Этого треда уже нет.
Это копия, сохраненная 15 июня 2017 года.

Скачать тред: только с превью, с превью и прикрепленными файлами.
Второй вариант может долго скачиваться. Файлы будут только в живых или недавно утонувших тредах. Подробнее

Если вам полезен архив М.Двача, пожертвуйте на оплату сервера.
105 Кб, 960x720
36 Кб, 740x308
39 Кб, 667x319
218 Кб, 672x1330
Математика, тред 56 Математика для нематематиков. #396075 В конец треда | Веб
Мемасы(в разработке):
http://pastebin.com/e38Yuj5V
Архивы тредов
http://pastebin.com/w1nJGYv4
Целый раздел по математике
https://2ch.hk/math/ (М)
Почти все математики ушли в /math. Но вы все равно можете обсудить ваши математические проблемы здесь. Серьёзно.
#2 #396113
Математики, решите предел пожалуйста.
lim n->inf (n*abs(sin n))
n - натуральные числа
396119396243
#3 #396119
>>396113
Предел не существует.
396155409364
#4 #396124
>>396075 (OP)
почему предыдущий тред не открывается в архиваче ?
396126404833
# OP #5 #396126
>>396124
Ты про этот >>392366 (OP)?
Вот он https://arhivach.org/thread/206244/
Если же про >>392615 (OP), то ветвь этого перехала в /math. Там и архивы.
404833
#6 #396131
Какого хуя cdf плеер даже папку установки назначить не даёт? Вроде солидная прога, а такой проёб.
#7 #396140
Решил таки зарыться в математику с головой до самых молодых ее писечек. Но хочу не просто заучивать формулы и теоремы, как меня пытались научить в инсте, а хочу аналитически ПОНЯТЬ каждую формулу и теорему, чтобы потом все эти знания активно экстраполировать на окружающий мир, пока меня не увезут с шизофренией. Насоветуй годных книжек, желательно по порядку, начиная с 1 курса, где авторы понимают о чем пишут, и объясняют физические и абстрактные смыслы всего утвержденного, а не просто интегралом машут.
#8 #396141
>>396140
Начни с Serge Lang "Basic Math", у него задания годные.
#9 #396142
>>396140
Винберг- Курс алгебры
Зорич-МА
с тебя хватит
396745
#10 #396145
Решил тут умножить 15х15 и соснул, выявил числа 75 и 15 что делать то с ними блеать 75 и 15
396146
#11 #396146
>>396145
1510+155=150+75=225
396147
#12 #396147
>>396146
Блядь, звёздочки.
15x10+15x5=150+75=225
0 Кб, 172x44
Пидор #13 #396148
Почему так?
396150396610
#14 #396150
>>396148
ВСЕ УВИДЕЛ СПС!
#15 #396155
>>396119
почему?
396159
#16 #396159
>>396155
abs(sin n) переодическая функция.
396162
#17 #396162
>>396159
И что?
396163
#18 #396163
>>396162
Неизвестно чему она равна на бесконечности.
https://www.google.ru/search?q=y+=+abs(x*sin(x))&ie=utf-8&oe=utf-8&client=firefox-b&gfe_rd=cr&ei=JEUnWJOaEeb37gTApoS4DA
Вот тебе график.
396164
#19 #396164
>>396163
И какое отношение этот график имеет к пределу функции при икс стремчщемся к бесконечности?
396166
#20 #396166
>>396164
Прокручивай график в право и смотри к чему стремися функция.
396168
#21 #396168
>>396166
Сколько бы я не прокручивал, до бесконечности я его не докручу.
396169
#22 #396169
>>396168
Но ты можешь заметить, что предела нет.
396170
#23 #396170
>>396169
Я могу построить на этой основе гипотезу. Но это ничего не доказывает.
396171
#24 #396171
>>396170
Ты же не думаешь, что я за тебя буду писать доказательство?
396173
#25 #396172
>>396140
Кострикин-Манин
396745
#26 #396173
>>396171
Я просто указал тебе на твою ошибку, только и всего.
396174
#27 #396174
>>396173
У меня не было ошибки. Я тебе не писал доказательство, а указывал путь к ниму.
396175
#28 #396175
>>396174
Вопрос задавал другой анон, не я. Я просто разместил объяву указал, что такое рассуждение некорректно.
396176
#29 #396176
>>396175
Я хотел направит его и объяснить смысл простым языком. Думая, что тот анон не очень умный. Ты ожидал более формального доказательства?
396177396178
#30 #396177
>>396176

>направить

#31 #396178
>>396176
Не, я не ожидал. Просто тот анон спросил, ты ему ответил, но твой способ хоть и дает правильный ответ в конкретной ситуации, в общем случае является некорректным. То есть тот анон мог бы подумать, что всегда так можно делать, а это не так, и в итоге получилось бы распространение ложного знания, увеличение энтропии и вообще баланс силы склонился бы на темную сторону. Да, я зануда. :3
396184396187396312
#32 #396184
>>396178
ну вот. а сейчас получилось что ты идешь нахуй
396186
#33 #396186
>>396184
У-тю-тю, обиделся!
396189
#34 #396187
>>396178
Просто лень было писать строгое формальное доказательство. Но почему это так, объяснить хотелось.
#35 #396189
>>396186
Просто лень было писать строгое формальное доказательство. Но чем это кончается, объяснить хотелось.
396190
#36 #396190
>>396189
Зачем ты коверкаешь мой пост?
396192
#37 #396192
>>396190
Хочу с тобой познакомиться.
396193
#38 #396193
>>396192
Ты тянка?
396194396197
#39 #396194
>>396193
петянка
#40 #396197
>>396193
Да. :3
396201
#41 #396201
>>396197
нет
396202
#42 #396202
>>396201
Пидора ответ.
#43 #396204

>читать книги вместо просмотра кханакадеми

396207
#44 #396207
>>396204

> 2016


> читать книги

sage #45 #396208
>>396075 (OP)

> пики в шапке на русском


> раковый вузиковый мемас, еще и без заглавной


Найс канцер-тред.
396211396212396619
#46 #396211
>>396208
да, ты зашел туда!
#47 #396212
>>396208

>пики в шапке на русском


Имбицил, мы на русской борде. Не нравится язык - пиздуй на форчан.

>раковый вузиковый мемас, еще и без заглавной


Мамка твоя раковая, этот мемас старше тебя. Увожай его, псина.

>Найс канцер-тред.


Это же каким надо быть дебилом, чтобы говорить о раке в 2016.

>sage


Полный даун.
396218
#48 #396218
>>396212

>Имбицил, мы на русской борде.


>имбицил


Если мы на русской борде, почему ты так плохо знаешь русский язык?
396221
#49 #396221
>>396218
Тебя ебёт чтоле?
396222
#50 #396222
>>396221
Где я - там Россия. Понимаю.
396225
#51 #396225
>>396222
Хуле ты приебался?
396234
#52 #396234
>>396225
ебло стяни валет
396240
#53 #396240
>>396234
Под шконарь, щенок.
#54 #396243
>>396113
Оно, конечно, очевидно, но если нужно формальное доказательство:
Возьмем отрезок от 2pik+1/4pi до 2 pik+3/4pi. Для любого целого k в этом промежутке будет хотя бы одно целое число m (промежуток больше 1). Синус этого целого числа m больше 1/корень(2). Видно, что выражение под пределом для подобных чисел стремится к бесконечности.
5575 Кб, 480x358
sage #55 #396299
Бездонна ли математика? Физика, по крайней мере, выяглядит ограниченной бытием наукой. Есть ньютон, стандартная модель, несколько квантовых теорий, описывающих микромир. Возможно, пройдёт двести лет, но микромир и макромир будет обьединён. Или не будет.
А что с математикой? Для профана она выглядит словно бездонный ковш.
Есть ли вообще какие то ограничения для математики? Существует ли некий принцип достаточности, где один математик говорит другому "Всё, тут хватит, потому что ты не создаёшь что-то новое, а пиздевово-гомотопово-хитровыебанно-через-12-мерный-континуум-поле-ландау рассматирваешь то, что ааранов и бом уже рассмотрели 20 лет назад", или "мойша, ты залез слишком глубоко, и вылез с другой стороны, хватит время тратить на бег по кругу", что то в таком духе? Просто если нет ограничений - то можно плодить сущности до бесконечности же, но интуитивно же очевидно, что где то это перестаёт иметь смысл, и всё переходит в теории о теориях о теориях о теориях, где вероятность ошибки достигает 100%.
396892
1 Кб, 199x104
#56 #396300
Олимпиадка 11 класс. Все иксы положительные. Кто может решить?
396311
#57 #396302
>>396075 (OP)
Есть неизвестное число, в двоичном коде:
01000101 или 11000101.
Есть результат умножения его на другое, известное, случайное число.
С результатом умножения, можно проводить операции сложения, вычитания, удвоения. Умножать и делить нельзя.

Вопрос: Возможно ли однозначно определить старший бит числа, и если нет, то почему?

Казалось бы, это может помочь:
01000101 + 10000000 = 11000101
11000101 - 10000000 = 01000101
Но знак зависит от старшего бита, опять же.
И 01000101 - 10000000 = отрицательное
а 11000101 + 10000000 = 101000101
399048
#58 #396311
>>396300
Ну смотри, начни с простого. Понятно, что все xi>1. Тогда ясно, что n<=8. Уже неплохо, но перебирать 8 вариантов нахуй надо. Давай заюзаем кое-что помощнее -- неравенство о средних. Тогда 9/n >= n/1 <=> n <= 3. Заебись. Случай n=1 тривиален, случай n=2 решается квадратным уравнением (реши сам). N=3 -- неравенство превращается в равенство, а это происходит только тогда, когда все xi равны между собой. Тогда легко понять, что x1=x2=x3=3.
396370396471
#59 #396312
>>396178
>>396178
Совушкин, это ты?
2 Кб, 275x34
#60 #396321
Я никогда не умел доказывать теоремы, а тут даже не знаю, как подступиться. Нужно доказать пикрелейтед с помощью алгебры логики. Как это делать?
396342
#61 #396342
>>396321
поэлементно распиши, че ты как этот
396351
#62 #396351
>>396342
А декартово произведение по аналогии с конъюнкцией что ли работает? Какие тут вообще формулы?
396353
#63 #396353
>>396351
В твоем случае это пары элементов множеств, так как множество индексов мощности 2.
#64 #396370
>>396311
Спасибо
4247 Кб, Webm
#65 #396404
396482
19 Кб, 675x86
#66 #396442
пик.

к чему стремится: (x - 1/n, x + 1/n) при n -> inf?
к {} или к {x}?

μ({x}) = μ(B_n), n -> inf
μ - мера
почему из этого следует, что μ({x}) = 0?
396460
#67 #396460
>>396442
бля, а ты совсем матана не знаешь? может нахуй эти ваши лебеги, если возникают вопросы, типа к чему стремится что-нибудь? стремится твоя интервал конечно к (x-0,х+0). причем сколь угодно близко.
396474
#68 #396465
Как развить правильное мышление для наук? Как перестать думать импульсивно, интуитивно... Точней, как обосновать то, что я думаю? Как стать богом логики? Как начать думать, как ученые? Какие нужны ментальные модели? Как развить абстрактное мышление? Как все это сделать? Помогите. Хотелось бы материал от трех годиков, ибо так себя чувствую.
396467396472
#69 #396467
>>396465
Непейвода прикладная логика pdf
396495397238
#71 #396472
>>396465
Пойа - Как решать задачу
sage #72 #396474
>>396460
понял.

B_n, n -> inf:
(x, x), пустой интервал {}.
B = пересечение B_n = {x}.

свойство непрерывности:
μ(B) = μ(B_n), n -> inf = μ(B_n, n -> inf)
тогда:
μ({x}) = μ({}).
#73 #396482
>>396404
Нам нужен CS-тред. Или это уже CS-тред. Если нет, то нам нужен CS-тред.
396483396484
#74 #396483
>>396482
Counter Strike или Compiter Science? Если второй, то я могу материалов для шапки собрать.
#75 #396484
>>396482
/pr/ же
#76 #396495
>>396467
Начал читать и ты не представляешь, как я тебе благодарен. Давно надо было реквестовать.
#77 #396527
Cуществуют ли несчётные точечные множества? Или эти два понятия противоположны по определению?
#78 #396531
Что делать, если вся математика улетучивается? Как удержать ее в голове? Как начать понимать все эти абстракции? И правильно ли будет привязать числа к геометрии? А геометрию как упихнуть? Может запилите свои кулстори?
396547
#79 #396532
Поясните за разные системы счисления и перевод из одной в другую. Все как-то запутано, может Вы поясните? Пожалуйста, прошу.
396570
#80 #396547
>>396531
представление числа в системе исчисления с основанием B это сумма произведений (линейная комбинация) положительного целого числа b в некой степени и чисел Ak больше нуля, но меньше b.
B=10,A1=1,A2=4,A3=8,A4=8
1488 = 10^31 + 10^24 + 10^18 + 10^08

предположим мы хотим найти вид числа x в системе исчисления меньшего основания y. Тогда поделим его с остатком. Получившийся остаток это самый маленький член в новом представлении (Почему?). Далее взглянем частное z от деления x=yz+k и поделим его на y. Взглянем на частное: z=yf+r. Число r это самый маленький член в представления числа z. Очевидно, можно продолжать деление, до тех пор, пока частное от него не станет меньше числа y, это число самый большой член в представления. (Почему?) Тогда подставим последнее уравнение в предыдущее, а получившееся уравнения в предыдущее, и так далее, получившееся уравнение будет вида x=y^(n)a+y^(n-1)b...+k, что и будет представлением исходного числа.

Пример. b1=10,b2=8, число 255
255/8=831 mod 7
31/8=8
3 mod 7
3/8= 0 mod 3
Следовательно: 255(10)=377(8). Проверим:
210^2+510^1+510^0=38^2+38^1+38^0
#81 #396570
>>396532
Например, 362= 310^2 + 610^1 + 210^0. В этом смысл позиционной системы записи чисел. Но видно, что степени десятки тут в принципе случайны - они от количества пальцев на руке - вместо десяти можно взять любое другое число больше 1 и посторить позиционную систему на нем. Проще всего строить на степенях двойки, тогда будут всего две цифры 0 и 1. Запись 1010 надо интерпретировать как 12^3+02^2+12^1+02^0 = 18+04+12+01= десяти в нормальной системе. Обратно тоже можно поднатужившись перевести. Для тройки будут цифры 0, 1, 2 и умножения на степени тройки и т.п. Например, 25 в троичной системе будет 25=29+23+11, то есть запись в троичной будет 221.
#82 #396593
Посоветуйте литературки по комплексному анализу. Англ или не англ не важно, понимаю и то и то. Желательно чтобы объясняли как для моранов вроде меня.
396602396604396667
#83 #396594
>>396075 (OP)
Поясните за монотонные и линейные булевы функции, я нихуя не понял из определения.
#84 #396601
>>396075 (OP)
1. x=asin(t); y=bcos(t); это эллипс?
2. x=asin(t); y=asin(t+f); это эллипс?
396671
#85 #396602
>>396593
шабат
а
б
а
т
396608
#86 #396604
>>396593
курант
у
р
а
н
т
#87 #396608
>>396602
ИМХО, учебник сложноват для вкатывания с 0.
#88 #396610
>>396148
Но Х^2 будет стремиться хуже Х..
А если записать это, заменив X на -1+А и при этом А устремить к 0, то получится фигня, т.е. -7, если я спьяну не обсчитался.
396612
#89 #396612
>>396610
ага

(2x^2-3x-5)'=4x-3
(x+1)'=1

(4*(-1)-3)/1=-7
#90 #396617
Подскажите как дифуру решить:
xy dx + (x+1) dy = 0
Переменные разделил, проинтегрировал, получил общий интеграл:
Ln |y| = Ln |x+1| - x - C
После потенциирования получается:
|y| = |x+1| exp(C) / exp(x)
Как правильно избавляться от модулей, и что делать с постоянной С, у которой область значений изменилась (не у самой постоянной, но у выражения, хз как правильно сказать).
396621
#91 #396619
>>396208
Похоже ты не послушал училку в школе.
#92 #396620
А если б Эйлер жил в наше время, он вошёл бы в тройку/десятку самых охуенных матанов, или всё-таки сейчас намного круче зубры есть?
1 Кб, 166x65
#93 #396621
>>396617
Это же линейное однородное уравнение, почему бы не решить его через интегрирующий фактор?
1 рядок - уравнение в стандартном линейном виде
2 рядок - интегрирующий фактор
3 рядок - общее решение
396627
#94 #396627
>>396621
Тема с интегрирующим множителем будет чуть дальше, пока думаю по порядку освоить все методы.
И как все-таки от модуля над (х+1) избавляемся? Знак от раскрытия модуля "вносим" в постоянную С?
396629
4 Кб, 221x57
36 Кб, 405x425
5 Кб, 221x70
#95 #396629
>>396627
Насколько я помню, у математиков в таких случаях принято расписывать случаи: решение с одной произвольной константой до точки разрыва и с другой - после. (1 картинка - ДУ, 2 -его поле направлений, 3 - общее решение). Если вкратце, то суть в том, что если ты двигался по одной ветке параболы до точки разрыва, то нет никаких оснований для того, чтобы не начать двигаться по ветке абсолютно другой параболы после точки разрыва. Лишь в этом случае это будет действительно общее решение. Но на самом деле при использовании ДУ в реальной жизни в подавляющем большинстве случаев эти модули ни во что не уперлись и с ними даже не заморачиваются. Главное не забывать о existence and uniqueness theorem и intervals of validity для линейных уравнений.
Кстати, в рашкинской программе иногда и не дают интегрирующие множители для линейных ДУ, а дают только для ДУ в неполных дифференциалах.
#96 #396631
Какие есть учебники по начальному курсу математики, по алгебре и геометрии?
Я знаю книжки туманова, понарина, алгебру и геометрию жидоШеня. Какие еще есть?

И еще желательно такую книжку в которой про линейную и высшую алгебру объясняли бы без гамалогий с тапалогеями и множаствами. Чтобы там показывали как вывели формулу для суммы первых n квадратов/кубов/тессарактов/т.д., про векторные поля, про кольца...
396888
41 Кб, 737x498
#97 #396640
Анон каким образом сделан вывод о том, что p - четное число? Я не понимат, помоги!
396641
#98 #396641
>>396640
А все, идите нахуй я и сам понял. Квадратный корень четного числа, всегда будет четным числом. И тоже с нечетным.
#99 #396648
Анон как ты относишься к числу Грэма?
#100 #396667
>>396593
На курсере есть курс. Analysis of a complex kind..
#101 #396670
бамп
#102 #396671
>>396601

>1. x=asin(t); y=bcos(t); это эллипс?


>2. x=asin(t); y=asin(t+f); это эллипс?

#103 #396728
Математики, а как Вы представляете математику у себя в голове? То есть, Вы используете какую-то привязку цифр к буквам и прочие мнемотехники? Как представляете себе бесконечность? Как производите вычисления? И т.д Расскажите, пожалуйста.
396730
#104 #396729
А если б был математик, умеющий видеть в 4 измерениях, он бы по комплексным функциям вообще лютый спец был и гипотезу Римана как нехуй делать доказал?
#105 #396730
>>396728
1. как учебник 3го класса
2. переменные представляю как недосказанные числа.
3. Столбиком, калькулятором, иногда разложением на "круглые" числа
396731
#106 #396731
>>396730
А можно подробней? Как вся эта математика фиксируется? Есть какая-то привязка к геометрическим фигурам и т.д
396732396733
#107 #396732
>>396731

>Есть какая-то привязка к геометрическим фигурам и т.д


только для физики
396739
#108 #396733
>>396731
фиксируется в виде формул
396739
#109 #396739
>>396732
Например?
>>396733
А формулы как представляются? Ты же не просто их зазубрил?
396740396741
#110 #396740
>>396739

>Например?


углы между направлениями движения, траектории тел, векторы скоростей.
396742
#111 #396741
>>396739

>А формулы как представляются? Ты же не просто их зазубрил?


как соотношения между числами, записанные на бумаге.
396742
#112 #396742
>>396740
>>396741
Да Вы как партизаны. Давайте подробней, пожалуйста. Ну, когда-то детям говорили, что число это типа количество яблок. Как Вы себе представляете это и уже вещи намного сложней. Вот Вы представляете себе бумагу, на ней числа, да?
396743396763
#113 #396743
>>396742
Кури "абстрактное мышление"
396744
#114 #396744
>>396743
Ну, епт, ничего нормального не находит. Лишь смотри на облачка да слова задом наперед читай.
#115 #396745
>>396142
>>396172
Спасибо
396911
#116 #396763
>>396742
Задавай вопрос в /матх лучше, тебе тут один школьник только отвечал. Хотя сам вопрос довольно интересный и сложный. Я не математик, так что мне вдвойне сложно судить об этом, но попытаюсь ответить. Если решаю какую-то конкретную задачу, то обычно просто пытаюсь пробовать разные известные мне методы решения, пока подсознание не найдет правильный. Это может работать просто как фокусировка на задаче, то есть смотрю на буквы и пробую с ними что-то сделать, а потом щелкает и становится понятно, либо как изменение представления, то есть я перевожу буквы из одной формы в другую, и на одной из форм щелкает и осознаешь, что вот это - то же самое, что вон то. У настоящих математиков это все происходит еще более интуитивно. С другой стороны, иногда бывает так, что ты терякшься и наоборот не понимаешь, что делаешь, не видишь общей картины, но поскольку ты знаешь формальные правила, то можешь делать следующий корректный шаг локально. То есть вот ты на пятом шаге, допустим, и не понимаешь уже, как это связано с тем, что было на первом шаге, но у тебя есть формальные инстоументы, с помощью которых ты можешь перейти от пятого шага к шестому и это будет гарантировано правильно - и уже после этого попытаться осознать, какая на самом деле связь между тем, что было на первом шаге и тем, что получилось. Вообще, математика - это в основном про то, как видеть, что разные вещи, не имеющие на первый взгляд ничего общего, являются на самом деле одним и тем же. В этом смысле это чистая, дистиллированная деятельность нашего мозга - в процессе эволюции он затачивался как раз под то, чтобы видеть паттерны и аналогии, и математика именно этим и занимается - только относительно не реального мира, а абстрактных, выдуманных систем. Да, ну а когда я не решаю данную задачу, а придумываю что-то свое, то тут гайдом в основном выступают эстетические категории - придуманное должно быть красивым, приятным, о нем должно быть приятно думать, оно должно нравиться. Опять же, у настоящих математиков это, наверное, выражено еще сильнее.

Вообще, погугли "интуиция и математика", Вейль кажется писал.
#116 #396763
>>396742
Задавай вопрос в /матх лучше, тебе тут один школьник только отвечал. Хотя сам вопрос довольно интересный и сложный. Я не математик, так что мне вдвойне сложно судить об этом, но попытаюсь ответить. Если решаю какую-то конкретную задачу, то обычно просто пытаюсь пробовать разные известные мне методы решения, пока подсознание не найдет правильный. Это может работать просто как фокусировка на задаче, то есть смотрю на буквы и пробую с ними что-то сделать, а потом щелкает и становится понятно, либо как изменение представления, то есть я перевожу буквы из одной формы в другую, и на одной из форм щелкает и осознаешь, что вот это - то же самое, что вон то. У настоящих математиков это все происходит еще более интуитивно. С другой стороны, иногда бывает так, что ты терякшься и наоборот не понимаешь, что делаешь, не видишь общей картины, но поскольку ты знаешь формальные правила, то можешь делать следующий корректный шаг локально. То есть вот ты на пятом шаге, допустим, и не понимаешь уже, как это связано с тем, что было на первом шаге, но у тебя есть формальные инстоументы, с помощью которых ты можешь перейти от пятого шага к шестому и это будет гарантировано правильно - и уже после этого попытаться осознать, какая на самом деле связь между тем, что было на первом шаге и тем, что получилось. Вообще, математика - это в основном про то, как видеть, что разные вещи, не имеющие на первый взгляд ничего общего, являются на самом деле одним и тем же. В этом смысле это чистая, дистиллированная деятельность нашего мозга - в процессе эволюции он затачивался как раз под то, чтобы видеть паттерны и аналогии, и математика именно этим и занимается - только относительно не реального мира, а абстрактных, выдуманных систем. Да, ну а когда я не решаю данную задачу, а придумываю что-то свое, то тут гайдом в основном выступают эстетические категории - придуманное должно быть красивым, приятным, о нем должно быть приятно думать, оно должно нравиться. Опять же, у настоящих математиков это, наверное, выражено еще сильнее.

Вообще, погугли "интуиция и математика", Вейль кажется писал.
#117 #396764
прямиком из мертвого /math:
сап. есть вопрос. дано: не переодический(к примеру) сигнал конечной длительности. с помощью преобразования Фурье мы получаем его спектр. сам вопрос: что такое спектр? его физический смысл? что он показывает? или у него нет абстракции? можно ли связать как то спектр с исходным сигналом путем аналогий или же фурье-преобразование это и есть единственная связь? можно ли надрочиться так, чтобы смотря на сигнал можно было видеть его спектр (хотя бы примерный?) спасибо заранее если кто нибудь прочитает этот высер
396774
#118 #396766
Посоны, у меня тупой вопрос.
Реально ли конструирование системы, в рамках которой число Пи будет равно %поебать_какому_числу%, т.е задано переменной?
#119 #396774
>>396764
Гугли мангу "анализ Фурье". Там поясняется в лайтовой форме (что даже такой конченный дебил как ты сможет понять).
396778
#120 #396778
>>396774
спасибо
#121 #396780
как из 3, 5, 8, 9 математическими действиями получить 10?
396781
#122 #396781
>>396780
5+5+(5-5)*(8+9)
396782
#123 #396782
>>396781

>5+5+(3-3)*(8+9)

72 Кб, 1280x960
#124 #396843
>>396075 (OP)

Аноны, помогите деграданту поделить 3 на 4,почему получается 0,75 а не 0,705 , разве мы не должны после того как спустится двойка добавлять 0, после семерки ?
396844397168402899
#125 #396844
>>396843
нет.
#126 #396888
>>396631
Бамп
396890
#127 #396890
>>396888
Сканави, Шень, Винберг, ???
397057
#128 #396892
>>396299
В самом деле, интересно. Но что касается физики, то, действительно, для нашей вселенной наверняка существует предел описательной математики. Однако, если допустить гипотезу мультивселенной, с бесконечным их числом, и как следствие со всевозможными законами физики, то и математика для каждой вселенной может быть совершенно различной, что приводит к отсутствию ограничений для нее.
396909
#129 #396909
>>396892
во всех вселенных одна математика. она не зависит от законов физики. нужно трёхмерное пространство описать? опишем. 11-мерное? опишем. сферическое? опишем.
396954397146
#130 #396911
>>396745
>>396745
ч-ч-ч, Винберг сильновато для тебя и не подробно

>с головы до писечек


Идеально подойдет двухтомник Городенцева, новый охуительный учебник, написанный для матфака ВШЭ (лучший матфак, после НМУ), к нему даже лекции есть на налекцию.рф , первую часть можно купить, вторая только в электронке.
Есть еще двухтомник Алгебра и Геометрия Зуланке, Онищика - отличнейшая вещь для новичка, если бы мне в свое время ее дали, мне в 3 раза проще было бы.
397056
#131 #396915
Посоветуйте чо почитать-порешать по дифференцированию и интегрированию, желательно с уклоном в физику.
Исчо, месяц назад в каком-то математика-треде мне посоветовали теорию чисел. Википедию читал, но нихуя не понял, почему тот анон назвал её дохуя интересной. Поясните тут.
397055
#132 #396923
Задача уровня /sci на отборе в Санофи

Вы владелец диагностической лаборатории для исследований биоматериалов в городе 500к населения. Для n различных пациентов, возраст и пол которых нормально распределены относительно всего населения, нужно сделать качественный анализ на ОДНО инфекционное заболевание. Каким образом можно снизить затраты на материалы и увеличить выручку, для всей лаборатории, сохранив точность результатов не ниже 99,85%. Во сколько раз возможно теоретически снизить затраты, в сравнении с методикой "1 пациент - 1 пробирка"?

Группы крови и резус-факторы у всех пациентов одинаковы.
396928
#133 #396926
Что первичней: кардиоида или жопа?
396968
#134 #396928
>>396923
Как точность определяешь? Т.е. что верный результат точно верный?
#135 #396954
>>396909
Утверждаешь, что в кокой-нибудь вселенной "жопа" не могут существовать законы физики, для описания которых не хватает современного математического аппарата? Разумеется, при допущении гипотизы мультиверсума.
397118
#136 #396967
Тут кто-нибудь шарит в тригонометрии?
#137 #396968
>>396926
Кардиоида
#138 #397045
>>396140
http://www.imperium.lenin.ru/~verbit/MATH/programma.html
Всё необходимое. Программа превосходит курс аспирантуры мехмата, лол. Но это не прикладная. Если хочешь в прикладную, то стоит смотреть лекции с лекториума мфти. Там всё необходимое для понимания, просто она прикладная.
398860
#139 #397055
>>396915
Steward Calculus
Spivak Calcuuls
#140 #397056
>>396911

>Идеально подойдет двухтоник Городенцева, новый охуительный учебник


>Городенцев


Тебя коником ебнуть или квадриком? Если под учебником подразумевается то, что выложено у него на сайте, но это вообще пиздос.
#141 #397057
>>396890
Слишком начально. Или у них есть книжка уровня тензоры для чайников?
#142 #397106
КТО-НИБУДЬ ШАРИТ В ТРИГОНОМЕТРИИ???77?7
397143
sage #143 #397118
>>396954
я даже не понимаю, с чего ты допустил такое предположение. математикой можно описать по крайней мере всё.
137 Кб, 400x173
#144 #397143
>>397106
Таблицы Брадиса.
397311
#145 #397146
>>396909
Алло блядь, математика -- это язык. Ты вообще понимаешь, что говоришь? Какая "одна" математика? Мы спокойно берём и меняем одну математику на другую, изменив аксиомы/правила вывода. Всё во всех вселенных, что возможно наблюдать, то можно и описать математически (хотя бы координаты, лол), ибо если мы это наблюдаем -- значит, мыслим. Если мыслим, значит, есть язык. А язык и есть математика, просто точную модель построить мы не можем в силу хаотичности аксиоматики и правил вывода. Но они есть.
397167
Аноним #146 #397166
какой КПД у Кварцевого генератора?
397174
#147 #397167
>>397146

>математика -- это язык


Это неверно. За такую философскую позицию бьют.
397197
#148 #397168
>>396843
3/4 = 0 + 3;
30/4 = 7 + 2;
2/4 = 0 + 2;
20/4 = 5 + 0;
397210
#149 #397174
>>397166
Вот никогда вопросом КПД автогенераторов не задавался. Ты какую-то сверхэкономичную схему проектируешь?
397382
#150 #397197
>>397167
Да и за фразу "бога нет" бьют, и за "сталин убийца" тоже (причём те же). Это не показатель неверности утверждений.
397208
#151 #397208
>>397197
То, что за эту позицию бьют, второстепенно. Первостепенно то, что она неверна.
397243
408 Кб, 2048x1152
#152 #397210
>>397168
Ты с разрядами попутал.
397218
137 Кб, 400x173
#153 #397218
>>397210
Ой, бля, сто нефти этому.
Я интеллектуально поднастроился
на ответ 0,705 не учитывая реал результата. Лол.

>2/4 = 0 + 2;


Не надо это.
Тут ноль должен был бы быть снесён сразу.

>20/4 = 5 + 0;



Поэтому, фикс:
3/4 = 0 + 3;
30/4 = 7 + 2;
20/4 = 5 + 0;
#154 #397238
>>396467
Ты охуел конструктивиста советовать?
397242397396
#155 #397242
>>397238
Я тебе больше скажу: он еще и ватник.

Ватник, конструктивист и удмурт.
397396
#156 #397243
>>397208
Ну если отвлечься от "математика это гранты, люди, институты, etc", то математика это класс формальных систем, что смело можно назвать языком.
397246
#157 #397246
>>397243
Математика не является формальной системой. Даже банально аксиомы ZFC мало кто может вспомнить.
397247401572
#158 #397247
>>397246
Мало-ли неграмотных на земле.

Математика имеет язык, а именно: набор применяемых символов - алфавит, и правила позволяющие отличать строчки языка от случайных наборов символов.
Этот язык в свою очередь имеет семантику, т.е. набор аксиом и правила вывода, что позволяет определять является ли строчка языка выводимой в этом формальном языке. Следовательно, математика является формальной системой.
Но это грубо, на самом деле подходов к формализации много, поэтому математика скорее совокупность формальных систем, из которых выбирают люди.
397251397504397505
0 Кб, 182x109
0 Кб, 182x109
#159 #397251
>>397247
Если бы математика была формальной системой, то люди, не знающие аксиом ZFC, не могли бы ею заниматься. Однако они занимаются.

>математика является формальной системой


Это всего лишь влажные мечты логиков. Нормальные математические доказательства не доводятся до уровня формальных систем. Например, рассмотрим следующую теорему с доказательством. Теорема о треугольнике, вписанном в прямоугольник.

Теорема. Площадь снаружи треугольника равна площади внутри треугольника.
Доказательство. Разрежем треугольник по пунктирной линии и увидим, что стороны треугольника рассекают каждую из частей ровно надвое.

Вот так и выглядит математика. Простые элегантные рассуждения, без всякого зубодробительного формализма.
397267
#160 #397267
>>397251
Почему именно аксиомы ZFC? Люди знают простейшие теоремы, правда чисто в синтаксическом исполнении, например, свойства сложения и умножения. Нетрудно убедиться, что эти теоремы по сути та же формальная система, с избыточным набором аксиом.

>Теорема. Площадь снаружи треугольника равна площади внутри треугольника.


Для данного рисунка?

>Доказательство. Разрежем треугольник по пунктирной линии и увидим, что стороны треугольника рассекают каждую из частей ровно надвое.


Это не доказательство, это вера в правильность рисунка. Используя такой способ можно придти к неверным выводам в случае с более сложными теоремами.

А если мы реально начнем доказывать, то надо начать хотя бы с того факта, что пунктирная линия делит центральный треугольник на два треугольника, а потом доказывать равенство каждого из них с соседним. Но это уже рассуждение высокого уровня. Ведь тут несколько раз применяются аксиомы равенства и постулат о параллельности, благодаря которым можно использовать утверждение о равных углах при параллельных прямых. "Видеть" это на деле просто обладать геометрической интуицией.

При этом для неевклидовых геометрий эта интуиция уже работать не будет. Придется свыкаться с другими аксиомами.
397276397277
#161 #397276
>>397267
А по-моему тот анон все правильно сказал, просто ты неправильно его понял.

Алсо, по-твоему математика = аксиоматический метод?
#162 #397277
>>397267
Доказательство - это сведение неочевидного утверждения к очевидным. Чем быстрее и неожиданнее это сведение, тем лучше. Хорошее доказательство должно вызывать смех и радость, как остроумная шутка. А формальные "доказательства" обычно вызывают смертную скуку.

Единственное, что мне не было очевидно в этом примере, - что площадь внутри треугольника равна площади снаружи. Я сделал так, что мне это стало очевидным. Всё, больше ничего тут делать не надо. Не нужно разговаривать ни о каких углах, ни использовать какие-то там "аксиомы равенства". Доказательство завершено.

И вот так везде. Нужно просто делать неочевидное очевидным. Именно в этом математика, и именно поэтому математика не является формальной теорией.

Формальные теории - это не более чем инструмент, которым работают в тех областях, где нет почти ничего очевидного. В областях, где очевидных вещей много, формальные теории не нужны.

Моё доказательство прочитали, и поняли, и приняли. Формальное "доказательство" тут больше уже не нужно. Писать формальное "доказательство" могут только прожженные бюрократы, которым место среди мертвых формуляров и пыльных печатей, а отнюдь не в живой, радостной науке. Да ведь если бы я запостил формальное "доказательство" этого факта, то его никто бы даже не прочитал.
397286397506
#163 #397286
>>397277
Опять же, детский сад. Давай точно так же докажи мне теорему Вариньона, чтобы мне очевидно стало. Да и в целом результат про треугольник весьма слабый, не обобщается на что-либо помимо евклидовых пространств.
397293
#164 #397293
>>397286

>детский сад


От того, что ты кого-то обзовёшь детсадовцем, его правота не изменится.

>докажи мне теорему Вариньона


Доказательство есть в вики, в одну строчку. Оно опирается на дистрибутивность векторного произведения относительно сложения, [a,b] + [a,c] = [a,b+c]. Интересной математики тут нет. И это доказательство тоже не является формальным, к слову.
397511
#165 #397298
математики, у меня к вам вопрос.
известно что число пи иррационально. но вот вопрос. оно иррационально только в системе по основанию 10 или во всех позиционных системах счисления? если взять систему счисления по основанию Пи то число пи перестанет быть иррациональным? есть ли такая система счисления по основанию которой все иррациональные числа перестанут быть иррациональными?

что если основание Пи х Е х 1\3 х (что там еще иррациональное) является универсальным? или из под него все остальные числа станут иррациональными?
397301
#166 #397301
>>397298
Иррациональность числа определяется независимо от какой-либо системы счисления. По-видимому, ты неправильно формулируешь вопрос, который тебя интересует. Спрашивай лучше, существуют ли такие позиционные системы отсчёта, в которых пи записывается периодической десятичной дробью.
#167 #397311
231 Кб, 1280x1443
#168 #397360
Может тут есть добрые, умные аноны, которые помогут мне подготовиться к пробному, чтобы меня не исключили из-за одной ебучей математики?
397385397386
Аноним #169 #397382
>>397174
нет. но по сути если брать колебания бесплатных вечных источников (микроколебания земли, дуновения ветра) то это как бы самая правильная схема вечного двигателя. ПОка существует земля и вращается , то это будет работать.

мне кажется это самая важна научная отрасль которую надо развивать. Автоколебания ключ к будущему
#170 #397385
>>397360
НУ ПОМОГИТЕ, Я ВАМ СКИНУ ПОСЛЕДНИЕ ДЕНЬГИ, ЕСЛИ НАПИШУ НА 4
#171 #397386
>>397360
Напиши на praverL_`yANUSmaill[^PUNCTUMr2&Cu, обсудим твою проблему

математик-кун
397461
#172 #397391
Крч. Я вывел формулу по доказательству того, что 1×1=3.
397434
#173 #397396
>>397238
>>397242
Что у них там? Конструктивизм?
Динамический реконструктивизм...
#174 #397416
Дарова, питурдики, мамкин криптограф из 5б вкатился, у меня вопрос (по вики):
Эйлер нашел простой многочлен n^2 -n +41, принимающий простые значения для n 0..40, какова вероятность математическая что это простое совпадение? Видел где то формулу вероятности числа быть простым что-то там x/ln(x) и доказано что не существует многочлена в одной переменной n, значение которого являются простые числа для челых значений n. Какого хуя?
#175 #397418
Вбросьти плиз вебмку где смонтажированные куски лекториума про гомологии с чуваком поехавшим в медицинской максе, оче прошу.
397426
#176 #397426
>>397418
Он не поехавший.
#177 #397434
>>397391
Ну давай
#178 #397461
>>397386
Отписал
#179 #397491
>>396075 (OP)
Анонимасы, посоветуйте, что почитать по основам теории алгоритмов (нужны универсальные алгоритмические модели и сложность алгоритмов).
За ответы заранее спасибо.
397503
#180 #397494
Есть бесконечная 3х мерная сетка, 2 объекта, 6 степеней свободы, каждый шаг отклоняет обе частицы на рандомную соседнюю клетку.
Какова вероятность, что 2 объекта встретятся на бесконечности времени?
397498
#181 #397498
>>397494
Стопроентная.
#182 #397503
#183 #397504
>>397247

>Математика имеет язык


>Математика — это язык

#184 #397505
>>397247

>Этот язык в свою очередь имеет семантику, т.е. набор аксиом и правила вывода,


Это не семантика.
397511
#185 #397506
>>397277
Большего собачьего бреда я не читал. "Смех" и "радость". Угу. А про математическую строгость (rigour) ты слышал, радостный?
397510
#186 #397510
>>397506
Право рассуждать о математической строгости имеют только математики. Ты не являешься математиком.
397523397563
#187 #397511
>>397293
Ну я имел в виду геометрическую теорему и ее следствие о площади

>>397505
Выводимо=имеет смысл.
397522
#188 #397522
>>397511

>Выводимо=имеет смысл.


Неверно.
397562
#189 #397523
>>397510
Мне неинтересно твоё мнение по поводу того кем я являюсь или не являюсь. Твоя позиция о сведении «неочевидного» к «очевидному» (остальное не выдерживает критики) — в корне ущербна, т.к. существуют «очевидные» высказывания (например предел последовательности) нуждающиеся в строгом определении, для дальнейшего развития теории. Не говоря уже о том, что «очевидность» — неопределённое понятие. Одним очам видно, другим нет.
397534
#190 #397534
>>397523
Понятия нуждаются в определении постольку, поскольку неочевидны, и ни на копейку больше. Толчком к появлению строгого понятия предела стал кризис анализа начала XIX века, когда журналы массово начали публиковать откровенный мусор, шизофазию, и понадобилось каким-то образом её заткнуть, - то есть когда предел перестал быть очевидным. Анализ флюксий и флюент не требовал строгого определения предела.

>Одним очам видно, другим нет.


Люди, которым очевидно одно и то же, образуют какой-нибудь заговор, семинар, научную школу или что-то подобное. Затем из этого заговора выделяются люди, которым что-то неочевидно, и образуют новый заговор. Это нормально, это прогресс.
397563
#191 #397535
>>396075 (OP)
подскажите пожалуйста какую-нибудь функцию, которая принадлежит пространству L_1[0,1], но не принадлежит пространству L_2[0,1],
397536
#192 #397536
>>397535
f(x) = 1/√x, если 0 < x <= 1
f(x) = 0 в противном случае
397547
#193 #397547
>>397536
спасибо.
8 Кб, 474x226
#194 #397549
>>396075 (OP)
помогите пожалуйста с примером из функционального анализа, есть 2 пространства сходящихся числовых последовательностей, X=l_2, и Y=l_1, и задано отображение Fx. Необходимо ответить, является ли заданное отображение:
a) непрерывным;
б)равномерно-непрерывным;
в)удовлетворяющим условию Липшица;
Объясните пожалуйста, как проверить все эти условия, определения всех этих понятий выписаны, но я не понимаю, как их проверить на практике,
объясните пожалуйста алгоритм решения такого задания.
#195 #397551
У нас есть 10 попыток назвать любую цифру от 0 до 9
Каждую попытку есть есть проигроишное число. Оно может быть любым получая таким образом 1010 комбинаций. Но так или иначе проигрышных цифр всего 10 по 1 на каждую попытку.
Какая наиболее не проигрышная стратегия. заранее выбрать 10 чисел, или каждый раз выбирать новое, или одинаково?
397552
#196 #397552
>>397551
По условию, все строчки из десяти цифр равновероятны. Поэтому однохуйственно.
#197 #397562
>>397522
Имеет смысл = существует конструктивный алгоритм для проверки смысла данного утверждения и он выдает правду. Для выводимости алгоритм очевиден.
397565
#198 #397563
>>397534
>>397510
Я тут просто мимо проходил и хотел сказать, что ты классный - оставайся, пожалуйста.
#199 #397565
>>397562
Ознакомься с понятиями soundness и completeness.
397570
#200 #397570
>>397565
Soundness обеспечивается самим построением. Completeness не знаю причем тут.
#201 #397580
Дрочил тут теорему Безу. И встрял на задании найти остаток от деления многочленов: x^100 - 2*x^51 + 1 на x^2 - 1

Не подскажете, как начать?
397581
#202 #397581
>>397580
Так. Я чуть-чуть подумал и решил представить первый многочлен в виде f(x^2) = x^50 - x^(51/2) + 1
Тогда остатком будет f(1), то есть один. Странно
397582
#203 #397582
>>397581
То есть нуль. Короче, странно
397583
#204 #397583
>>397582
А, все. Всем спасибо
397585
#205 #397585
>>397583
Обращайся, дружище.
#206 #397592
Может кто-нибудь помочь с заданиями по комплексным числам?
В первом надо вычислить, во втором найти геометрическое место точек на комплексной области, удовлетворяющих этим условиям
397593397618
#208 #397618
>>397592
в любой непонятной ситуации домножай на сопряженное!
3598 Кб, Webm
#209 #397666
https://www.youtube.com/watch?v=yQNeHG3jcZU
Спешите ознакомиться с реальной математикой.
#210 #397714
Есть математик-куны? Напрягли меня тут докладом по теме: "Решение интегралов вида ∫cosαx⋅cosβx dx". Гугл ничего путного не выдает. Может кто мог бы помочь?
397718402936
sage #211 #397718
>>397714
заглянуть в википедию например.
4 Кб, 466x35
#212 #397785
Как понимать?
397810397813
#213 #397810
>>397785
пусть он конечен... тогда
#214 #397812
Я давно не заходил в мат. тред, подскажите аноны, здесь ещё тот кун, который доказывал несовместимость аксиом выбора и детерминированности? И в каком треде это было, хотя бы примерно? Помню, что доказывал он не опираясь на терминологию из теорию игр. Хочется освежить в памяти.
#215 #397813
>>397785
Множество бесконечно, если есть биекция между всем множеством и некоторым его подмножеством. Или что именно тебе не понятно?
2 Кб, 132x57
#216 #397822
Поясните за такой предел. У меня получилось 0, у сайтов разные варианты ответов: 00, 1\е и е^-1
Пздц... Ведь тут вроде бы можно даже не считать, и так понятно что в числитель будет бесконечно уменьшаться, а значит будет 0 !! Ну, анончики, обьясните мне, или решение подкиньте, правильное. Мне заочнику надо сдать это, а в феврале математики уже не будет в моей учебе.
#217 #397823
>>397822
выражение в скобках при 10 будет 19/20, при 100 199/200, а значит стремится к единице. Показатель стремится к бесконечности. Предсказать результат нельзя, т.к. числа стремящиеся к единице в бесконечной степени могут дать любой результат.

Обычно в этом случае поступают так: представляет x^y в виде e^ylnx, и вычисляют предел уже этого выражения.
397862
sage #218 #397825
>>397822
В ун/ уёбуй.
#219 #397834
>>397822
exp(-1) ответ

(2x-1)/(2x)=1-1/(2x).Предел этой фигни в степени 2х будет exp(-1), а предел скобки в третьей степени будет 1.
397862
#220 #397838
А если б гипотезу Римана 100 лет назад доказали, по ней всё равно бы сотни работ до сих пор делали?
#221 #397848
>>397822
у меня ответ = 3

(1\2(1+2x)x^2x+3)=3
397851397862
#222 #397851
>>397848
Докажи по определению, предъявляй эпсилон.
#223 #397857
>>397822
вот есть у тебя эта твоя фигня. то что в скобках можно расписать как разность 2х\2х- 1\2х. 1\2х при устремлении к бесконечности будет бесконечно малой и хуй на неё забьем. а 2x\2x=1. поэтому твой пердел можно переписать как единичку в бесконечной степени. единичка в любой степени будет единичкой. т.е. ответ 1. пруф ми вронг.
397858397862
#224 #397858
>>397857
лан, не надо пруфать. вспомнил про второй замечательный пердел. давно просто всё это говно было.
6 Кб, 288x94
#225 #397862
>>397822
это же замечательный lim ! Решение на пике, ответ e^1

Не слушай этих петухов: >>397823 >>397834 >>397857 , они матан не знают, студентота 1 курс

>>397848
ты всё правильно сделал, но только если бы Х стремился к 0

мимо_преподаватель_этих_ваших_втузов
397879
#226 #397865
Привет и прощайте аноны.
26 лет.
Хотел в науку.
Нет образования.
Старый.
Вскрываю вены
397866397874
#227 #397866
>>397865
я это ты, чувак. Давай выпилимся вместе. Какой город?
#228 #397874
>>397865
Это никого не волнует, маня. Съеби.
397876
#229 #397876
>>397874
меня волнует
#230 #397879
>>397862
охуенный знаток. Предъявляй эпсилон давай.

hint: функция не достигает и 1/2, не то что e
#231 #397889
Привет, пацаны. Гуманитарий в треде.
Хочу угореть по геймдизайну, работаю в близкой сфере.
Пишу тексты, персонажей, квесты, все дела.
Но для того, чтобы работать геймдизайнером, - нужно знать тервер и статистику.

Можно ли задрочить эти сферы, не отвлекаясь на другую математику?
Насколько сложно для ньюфага, который особо не горит въезжать в математику, но нужно для работы.
Что посоветуете почитать?
397893
#232 #397893
>>397889
Вообще-то тебе интегралы еще надо брать. Короче, программу первого курса бери. Если в школе у тебя не было фундаментальных проблем с математикой, то несложно, вопрос только во времени.

Кстати, на курсере есть и матан, и статистика, и микроэкономика и вообще все, что тебе нужно.
4 Кб, 662x119
#233 #397895
Правильно доказал хоть?
397897
#234 #397897
>>397895
Надо было тогда уж начать так: "пусть икс принадлежит <левой части>. Тогда...", а потом: "пусть икс принадлежит правой части, тогда..." - и каждый переход сопровождать ссылкой на обоснование справедливости перехода.
1 Кб, 134x36
#235 #397917
Лолшто это? логическая операция на множествах. Пруверы изучай. Не умеешь в прувер не умеешь в доказательство.
#236 #397922
Внимание задача
Продавец продаёт шапку. Стоит 10 р. Подходит покупатель, меряет и согласен взять, но у него есть только банкнота 25 р. Продавец отсылает мальчика с этими 25 р. к соседке разменять. Мальчик прибегает, и отдаёт 10+10+5. Продавец отдаёт шапку и сдачу 15 руб. Через какое-то время приходит соседка и говорит, что 25 р. фальшивые, требует отдать ей деньги. Ну что делать, продавец лезет в кассу и возвращает ей деньги. На сколько обманули продавца?
397923397925397947
#237 #397923
>>397922
35
Шапка 10 р + 25 р возврат соседки.
#238 #397924
Хотя, 10 рублей ему вовзращают за шапку. Поэтому 25 рублей.
#239 #397925
>>397922
Шапка(10)+15р покупателю-наебателю + 25 соседке итого 50 рублей
#240 #397947
>>397922
15 рублей
#241 #397951
Можно ли все доказательства переделать так, чтоб они состояли из одних математических знаков? Или теорема о неполноте по сути о невозможности этого?
397952397954
#242 #397952
>>397951
Можно и саму теорему о неполноте составить из мат знаков и даже небо, даже алдаха! Мат знаки это те же слова.
#243 #397954
>>397951
Мат символы это те же самые слова. Важно не то какой закорючкой ты выражаешь слово, а то какой смысл, какую структуру порядок и отношение ты в это слово вкладываешь. Вообще математики используют символы только для того, чтобы подчеркнуть свою элитарность. Чтобы для не сведущего человека вся запись казалась непостижимой тарабарщиной. Тем самым у человека начинает складываться образ математиков, как каких то гениев недостижимого уровня. Сами математики в глазах быдла всячески стараются этот образ, миф поддерживать.
#244 #397958
Двачики,кто может и хочет помочь пройти тест по математике на экзам?Там точно есть пределы,матрицы,логарифмы и т.д
Сам я точно не пройду а от этого зависит получение зачета,в общем помогите плиз
133 Кб, 400x400
26 Кб, 660x440
#245 #397967
Двач, выручай! Есть ли в математике какое-нибудь название для фигур вида пикрелейтед(насколько я понял, между ними есть что-то общее, если нет - поправьте)? Пока нашел только две картинки, другое не могу представить, а надо нарисовать. Вот как гуглить, не знаю.
397968397969398013
#246 #397968
>>397967
Добавлю, что надо, чтобы они были трёхмерными. Если знаете, где, подскажите, плиз.
#247 #397969
>>397967
Неориентируемая поверхность, мб
398013
#248 #397973
Как грамотно доказать, что неравенство не имеет решений при любом натуральном n > 1?
2^n > n^1000

Пробовал пока что логарифмировать и решать неравенство в лоб, но не вышло чего-то хорошего. Предполагается, что число e и соответствующий замечательный предел решающий пока не знает.
#249 #397974
>>397973
Оно имеет решения.
397975398017
#250 #397975
>>397974
Например?
398017
#251 #397978
>>397973
2^2=4>2^1000
WHAT?!!!!!!!!!!
398017398040
#252 #398013
>>397967
>>397969
Кажется, нашёл. Всё оказалось проще простого: это УЗЛЫ.
#253 #398017
>>397978
>>397975
>>397974
>>397973

Разобрался, спасибо. Хотя, конечно, задание подразумевало тупой перебор, но потом на досуге подумаю о решении неравенства в целом. Спасибо.
398040
#254 #398035
>>396075 (OP)
Как называется задача вида:
\int_0^inf f(x-c)dx=k, где k и c действительный числа?
#255 #398040
>>397978
Чего блядь?

>>398017
Задание не подразумевало перебор, гугли orders of growth.
398049
#256 #398049
>>398040
"Укажите такое n, при котором 2^n > n^1000"
4 Кб, 222x86
#257 #398072
Двач, помоги найти производную(y`) от этого.
398075
76 Кб, 456x750
#259 #398082
Когда учился на первых курсах у нас был матан, который мне было приятно учить.
И тогда у меня варила голова, я хоть чего-то соображал, мышление было ясным и острым.
Потом матан кончился и я за год-другой без математики превратился в туповатого дауна.

Посоветуйте раздел математики/учебник/задачник, который поможет хорошо размять мозги, привести ум в порядок, только не слишком сложный, чтобы я не забил на всё это дело, но и не слишком простой, чтобы был интерес.
398092399211
#260 #398087
короче есть поле для дартса

чувак 100% кидает в поле, какой шанс попасть во внешний круг, если Радиус внутреннеого круга в 2 раза меньше чем внешний?
398091
sage #261 #398091
>>398087
вероятность = (мера искомого множества)/(мера всего множества)
#262 #398092
>>398082
Очевидная теория множеств очевидна. У неё уровень сложности может варьироваться от детсадовского до космического, а за нежелание пользоваться умом мгновенно прилетает лопатой по лбу.
#263 #398124
Почему никто не уделяет внимания процессу "думания"
Все эти гильберты, формалисты, теоремы о неполноте, интуиционизм и логицисты все это сранные философы,
в то время как "И для специалистов, и для любителей не
философия, а именно активные занятия самой математикой смогут дать
ответ на вопрос: Что такое математика?"
( из предисловия Куранта Что такое математика)
короче Пойа "как решать задачу"
и "думай математически" Мейсона
А дальше "Тому, что в математике действительно необходимо и полезно знать, можно выучиться в шесть месяцев; всё же остальное составляет только предмет любопытства."
398189
#264 #398162
>>396075 (OP)
Vfyzvfnbrb dhjlt eikb? f lfeys c ntjhbzvb vyj;tcnd jcnfkbcm/
Pft,bcm
sage #265 #398189
>>398124
математика это комплекс методов. очень разнообразных.
398196
#266 #398196
>>398189
Надеюсь ты не подразумеваешь что это мне как то противоречит?
#267 #398831
Подскажите статейку по теореме Гёделя о неполноте.
398832398854
#268 #398832
>>398831
Gödel, Kurt. "Über formal unentscheidbare Sätze der Principia Mathematica und verwandter Systeme I".
#269 #398854
>>398831
Q: Правда ли, что Гёдель доказал, что мир невозможно познать?
A: Нет. Если выразить суть теоремы о неполноте простым языком, она утверждает, что человеческое мышление (и, в частности, математическое доказательство) - это существенно творческий процесс, и его нельзя свести к так называемой "китайской комнате", то есть к формальному исполнению некоторого алгоритма. Гёдель построил логическую формулу, выражающую свою собственную формальную (!) недоказуемость. Если бы эта формула была доказуема, это привело бы к противоречию в теории. Поэтому она недоказуема, а значит, верна, поскольку сама по себе именно это и утверждает. Кажущийся парадокс (мы доказали истинность недоказуемого утверждения) возникает из-за фундаментальной разницы между алгоритмическим (формальным) и семантическим (творческим) способом рассуждений - второй оказывается существенно богаче и мощнее. Вот и всё. Ничего сложного, если не брать технические детали построения самой формулы, которые довольно занудны.

Q: Значит ли это, что у человека есть душа, а искусственный интеллект невозможно построить?
A: Не значит. В процессе Гёдель сделал множество предположений, совершенно естественных для того, что мы обычно понимаем под формальной алгоритмической процедурой, но которые совершенно неверны для, скажем, человеческого мозга. К примеру, ключевым предположением является алгоритмическая разрешимость множества КОРРЕКТНЫХ формальных доказательств, то есть предполагается существование алгоритма (в строгом смысле, машины Тьюринга, например), способного отличать верное рассуждение от неверного. Это, скорее всего, не так, поскольку естественный язык позволяет формулировать утверждения, не являющиеся ни истинными, ни ложными. На простом языке это означает, что способность ошибаться и "тупить" является неотъемлемым свойством творческого процесса, и не ошибается только безмозглый автомат. Кроме того, Гёдель предположил конечность алфавита (и, следовательно, счётность слов в этом алфавите), в то время как множество состояний мозга, скорее всего, несчётно.

Q: Но зачем нам тогда формальные системы и аксиомы, если они такие плохие?
A: Затем, что формализованное доказательство, как уже было сказано, допускает алгоритмическую проверку своей корректности. Это позволяет практически полностью исключить из математики влияние авторитета и добиться объективности получаемых результатов. Формула, полученная Гёделем, строится для конкретной формальной системы, то есть для конкретных аксиом и правил вывода. В другой системе та же формула может оказаться прекрасным образом доказуемой, и никакого противоречия уже не будет. Однако теорема о неполноте говорит нам, что полностью свести математику к формальному языку, раз и навсегда задав систему аксиом и правил вывода, у нас не получится. Основания математики так же подвижны, как основания любой другой естественной науки. Она растёт и вверх, и вниз.
#269 #398854
>>398831
Q: Правда ли, что Гёдель доказал, что мир невозможно познать?
A: Нет. Если выразить суть теоремы о неполноте простым языком, она утверждает, что человеческое мышление (и, в частности, математическое доказательство) - это существенно творческий процесс, и его нельзя свести к так называемой "китайской комнате", то есть к формальному исполнению некоторого алгоритма. Гёдель построил логическую формулу, выражающую свою собственную формальную (!) недоказуемость. Если бы эта формула была доказуема, это привело бы к противоречию в теории. Поэтому она недоказуема, а значит, верна, поскольку сама по себе именно это и утверждает. Кажущийся парадокс (мы доказали истинность недоказуемого утверждения) возникает из-за фундаментальной разницы между алгоритмическим (формальным) и семантическим (творческим) способом рассуждений - второй оказывается существенно богаче и мощнее. Вот и всё. Ничего сложного, если не брать технические детали построения самой формулы, которые довольно занудны.

Q: Значит ли это, что у человека есть душа, а искусственный интеллект невозможно построить?
A: Не значит. В процессе Гёдель сделал множество предположений, совершенно естественных для того, что мы обычно понимаем под формальной алгоритмической процедурой, но которые совершенно неверны для, скажем, человеческого мозга. К примеру, ключевым предположением является алгоритмическая разрешимость множества КОРРЕКТНЫХ формальных доказательств, то есть предполагается существование алгоритма (в строгом смысле, машины Тьюринга, например), способного отличать верное рассуждение от неверного. Это, скорее всего, не так, поскольку естественный язык позволяет формулировать утверждения, не являющиеся ни истинными, ни ложными. На простом языке это означает, что способность ошибаться и "тупить" является неотъемлемым свойством творческого процесса, и не ошибается только безмозглый автомат. Кроме того, Гёдель предположил конечность алфавита (и, следовательно, счётность слов в этом алфавите), в то время как множество состояний мозга, скорее всего, несчётно.

Q: Но зачем нам тогда формальные системы и аксиомы, если они такие плохие?
A: Затем, что формализованное доказательство, как уже было сказано, допускает алгоритмическую проверку своей корректности. Это позволяет практически полностью исключить из математики влияние авторитета и добиться объективности получаемых результатов. Формула, полученная Гёделем, строится для конкретной формальной системы, то есть для конкретных аксиом и правил вывода. В другой системе та же формула может оказаться прекрасным образом доказуемой, и никакого противоречия уже не будет. Однако теорема о неполноте говорит нам, что полностью свести математику к формальному языку, раз и навсегда задав систему аксиом и правил вывода, у нас не получится. Основания математики так же подвижны, как основания любой другой естественной науки. Она растёт и вверх, и вниз.
#270 #398860
>>397045

>Нет теорвера и статистики


Ну с этим разве что в ПТУ.
71 Кб, 830x1261
#271 #398902
Сап, боги-математики. Если решите хоть один пример с расписанным ходом решения, я буду благодарен.
398903398904
#272 #398903
>>398902
Тут домашку не делают.
#273 #398904
>>398902
Одно задание сотка рублей.
398915
10 Кб, 570x411
20 Кб, 589x329
#274 #398910
>>396075 (OP)
Что я делаю не так?
Многодимензиональный куб #275 #398913
Народ, что делать?
Есть d-мерный куб (длина стороны - a). У него соответственно 2^d углов. Столько же d-мерных шаров с радиусом a/4 мы хотим наказать и поставить в угол так, чтоб они были полностью внутри куба. И вот вопрос. С какого d можно впихнуть ещё один шар в середину куба, чтоб он пробивался через куб? Помогите советом.
398960
13 Кб, 1037x389
#276 #398915
>>398904
Ладно, хотя бы скажите бесплатно, правильно тут всё?
398916
1 Кб, 156x89
#277 #398916
>>398915
Вот задание
#278 #398960
>>398913
>>398913
Вроде, с N=4 уже должно бы влазить. По симметрии задачи, надо рассмотреть диагональ x1=x2=...xN. Взять а=4, тогда круг из центра будет сечь линию при всех x=1/sqrt(N). Посчитав расстояние от этой точки до (1,1,...1) получим искомое.
398976
#279 #398976
>>398960
я нашла, что только с d=10 он выпирает. d=9 соприкосновение со стенками. Но доказательства нет...
399037
#280 #399037
>>398976
Я просто не понял вопроса.
Если катать шар по шару, то центр подвижного идет по шару с радиусом 2 (оба единичные для простоты ). Тогда если сделать вокруг точек (+-1, +-1 …. ) шары двойного радиуса, вопрос в том, будет ли соответствущая полость иметь выход к поверхности. Возможно, прокатит сначала надуть шары до такого радиуса, чтобы осталась лишь маленькая полость в центре, а потом сдувать помаленьку? Вероятно, протекание полости на грань будет при бифкркационном значении радиуса – когда какие-то пары шаров начинают касаться, а не пересекаться. Бифуркаций при сдувании явно будет несколько ( порядка N в натуре ). Может из этого что-то можно выдоить?
399054
Sage #281 #399048
>>396302
0+0=0
1+0=1
0+1=1
1+1=0
#282 #399054
>>399037
смогла вырешать очень просто через теорему пифагора )))) столько нервов потрачено на простейшую фигню.
399065399068
25 Кб, 603x439
#283 #399055
В чем ошибка?
399056399057399063
#284 #399056
>>399055
в днк
умри
#285 #399057
>>399055
В первом же шаге - копейки должны быть в квадрате под корнем.
#286 #399063
>>399055
Сначала приведём всё к единым единицам и будем считать только копейки.
Это просто в данном случае, ибо в рубле 100 копеек, потом слово "рубля" без вреда для здоровья заменяем на " 100"
sqrt(1/4
100) = 5 копеек
1/2 * 100 = 50 копеек
399064
#287 #399064
>>399063
"x 100"
sqrt(1/4 x 100) = 5 копеек
1/2 x 100 = 50 копеек
Где x = умножение.
#288 #399065
>>399054
А нафиг вообще эта задачка?
399068
#289 #399068
>>399065
Мне тоже интересно.

>>399054
Расскажи, будь ласка.
12 Кб, 1557x840
#290 #399080
Сап, имею вот такой предел. С числителем все понятно, просто на сопряженную домножить нужно. А вот как преобразовать знаменатель? Там по идее нужно эквивалент сделать? А как?
399102
#291 #399102
>>399080
изи же, выносишь -2, получаешь a^x - 1 ~ xlna
399155
#292 #399155
>>399102
сенкс
#293 #399211
>>398082

>Посоветуйте раздел математики/учебник/задачник, который поможет хорошо размять мозги, привести ум в порядок, только не слишком сложный, чтобы я не забил на всё это дело, но и не слишком простой, чтобы был интерес.



"Полный сборник задач по математике для поступающих в вузы. Группа повышенной сложности" - https://www.livelib.ru/author/385499/latest
Cтудентка смогла найти сбежавшего брата с помощью анализа социальной сети #294 #399215
https://www.hse.ru/news/edu/198594848.html

Достаточно стандартная семейная история — подросток сбежал из дома, написав смску: «Я сюда больше не вернусь, можете даже не пытаться меня найти» и, естественно, отключив телефон. Найти его смогла сестра, используя знания, полученные во время учебы на программе «Прикладная математика и информатика». Эта история также показывает, что могут рассказать соцсети о своих пользователях понимающему человеку.

Никто не знал, к кому мог уехать беглец — родители были полностью уверены, что молодой человек близок лишь со своими одноклассниками. Но тех, кого знали, обзвонили, и ни у кого из них он не обнаружился. Был еще один путь — поиск по сети Вконтакте. Именно этим и занялась сестра беглеца, которую из этических соображений мы назовем инициалом Л.

«Но друзей у него там было слишком много, чтобы проверить каждого, — вспоминает она. — Как посчитали статистики, в среднем у каждого пользователя социальной сети около 150 взаимных подписчиков, в то время как в реальной жизни круг общения в пять раз меньше виртуального. Нужно было искать другой способ. В курсе «Комбинаторика» мы в том числе писали программу обработки, анализа и визуализации сети друзей ВКонтакте. Я решила посмотреть, смогу ли я что-то полезное извлечь из этого анализа, поскольку любая информация приветствовалась».

+++

Все сделанные аналитически выводы позже подтвердились показаниями сбежавшего брата. Сам он до сих пор считает, что его сестра владеет какой-то математической магией.

«На этом этапе я наконец решила начать переписку и связалась с личностями А, Е и F, — рассказывает Л. — Это было рискованно, но стоило попытаться. Личность F в итоге оказалась бесполезна, хотя во время общения продемонстрировала открыто враждебную позицию. Личность A предпочла меня проигнорировать. Личность E некоторое время утверждала, что ничего не знает, но после, в общем-то, безосновательного, но очень уверенного напора все же признала, что брат находится в безопасности неподалеку. После этого было уже нетрудно опосредованно уговорить его выйти на связь сначала со мной, а потом уже и с родителями».

К этому моменту родители уже поехали подавать заявление о розыске в органы внутренних дел, захватив с собой «черный список» из этих трех личностей. Предполагалось, что если за выходные брат не появится, с них можно будет начать поиски. Но до этого не дошло, молодой человек сам вернулся домой.

После завершения этой истории граф его социальных связей выглядел так. Голубые точки — это друзья из новой школы с уклоном в сторону развития творческого потенциала учеников и патриотического воспитания, куда его перевели. На графе они не пересекаются с остальными. Участники же старых зеленого и фиолетового кластеров, которые на этом рисунке еще присутствуют, вскоре были удалены из друзей.

Преподаватели департамента анализа данных и искусственного интеллекта благодарны Л. за то, что она поделилась с ними своей историей. «Огромной радостью для педагога является не только успех его учеников, но и применение знаний на практике, особенно в таком важном деле, как спасение жизни ребенка», — говорит научный руководитель Л. и автор курса по комбинаторике Илья Макаров. — Я уверен, что этот пример заставит многих задуматься о том, сколько личной информации на самом деле хранится в социальных сетях и как обученные профессионалы могут использовать эти данные для предотвращения подобных случаев в будущем».

Описанные выше методы анализа социальной сети в Вышке можно изучить в рамках дисциплин и проектов на магистерских программах «Науки о данных» и «Прикладная статистика с методами сетевого анализа», они также затрагиваются на программе «Журналистика данных». Для всех магистрантов 1 года обучения любых других программ доступен вводный курс «Social Network Analysis» из цикла МАГоЛЕГО, а бакалавры могут взять курс по выбору «Комбинаторика».
Cтудентка смогла найти сбежавшего брата с помощью анализа социальной сети #294 #399215
https://www.hse.ru/news/edu/198594848.html

Достаточно стандартная семейная история — подросток сбежал из дома, написав смску: «Я сюда больше не вернусь, можете даже не пытаться меня найти» и, естественно, отключив телефон. Найти его смогла сестра, используя знания, полученные во время учебы на программе «Прикладная математика и информатика». Эта история также показывает, что могут рассказать соцсети о своих пользователях понимающему человеку.

Никто не знал, к кому мог уехать беглец — родители были полностью уверены, что молодой человек близок лишь со своими одноклассниками. Но тех, кого знали, обзвонили, и ни у кого из них он не обнаружился. Был еще один путь — поиск по сети Вконтакте. Именно этим и занялась сестра беглеца, которую из этических соображений мы назовем инициалом Л.

«Но друзей у него там было слишком много, чтобы проверить каждого, — вспоминает она. — Как посчитали статистики, в среднем у каждого пользователя социальной сети около 150 взаимных подписчиков, в то время как в реальной жизни круг общения в пять раз меньше виртуального. Нужно было искать другой способ. В курсе «Комбинаторика» мы в том числе писали программу обработки, анализа и визуализации сети друзей ВКонтакте. Я решила посмотреть, смогу ли я что-то полезное извлечь из этого анализа, поскольку любая информация приветствовалась».

+++

Все сделанные аналитически выводы позже подтвердились показаниями сбежавшего брата. Сам он до сих пор считает, что его сестра владеет какой-то математической магией.

«На этом этапе я наконец решила начать переписку и связалась с личностями А, Е и F, — рассказывает Л. — Это было рискованно, но стоило попытаться. Личность F в итоге оказалась бесполезна, хотя во время общения продемонстрировала открыто враждебную позицию. Личность A предпочла меня проигнорировать. Личность E некоторое время утверждала, что ничего не знает, но после, в общем-то, безосновательного, но очень уверенного напора все же признала, что брат находится в безопасности неподалеку. После этого было уже нетрудно опосредованно уговорить его выйти на связь сначала со мной, а потом уже и с родителями».

К этому моменту родители уже поехали подавать заявление о розыске в органы внутренних дел, захватив с собой «черный список» из этих трех личностей. Предполагалось, что если за выходные брат не появится, с них можно будет начать поиски. Но до этого не дошло, молодой человек сам вернулся домой.

После завершения этой истории граф его социальных связей выглядел так. Голубые точки — это друзья из новой школы с уклоном в сторону развития творческого потенциала учеников и патриотического воспитания, куда его перевели. На графе они не пересекаются с остальными. Участники же старых зеленого и фиолетового кластеров, которые на этом рисунке еще присутствуют, вскоре были удалены из друзей.

Преподаватели департамента анализа данных и искусственного интеллекта благодарны Л. за то, что она поделилась с ними своей историей. «Огромной радостью для педагога является не только успех его учеников, но и применение знаний на практике, особенно в таком важном деле, как спасение жизни ребенка», — говорит научный руководитель Л. и автор курса по комбинаторике Илья Макаров. — Я уверен, что этот пример заставит многих задуматься о том, сколько личной информации на самом деле хранится в социальных сетях и как обученные профессионалы могут использовать эти данные для предотвращения подобных случаев в будущем».

Описанные выше методы анализа социальной сети в Вышке можно изучить в рамках дисциплин и проектов на магистерских программах «Науки о данных» и «Прикладная статистика с методами сетевого анализа», они также затрагиваются на программе «Журналистика данных». Для всех магистрантов 1 года обучения любых других программ доступен вводный курс «Social Network Analysis» из цикла МАГоЛЕГО, а бакалавры могут взять курс по выбору «Комбинаторика».
72 Кб, 450x450
#295 #399259
Смысл числа Пи ведь в отношении длины окружности к её диаметру?

А с чего тогда математики уверены, что все формулы для вычисления этого числа без привязки к окружности являются именно числом Пи, а не более изощрённой его аппроксимацией, как 22/7, только более точной?

Его можно через пределы, через ряды, через бесконечные дроби, через интеграл, методом Монте-карло посчитать, но многие из этих методов и близко не связаны с окружностью по крайней мере, на первый взгляд.
С чего тогда все решили, что все эти методы расчёта даёт именно то самое число пи, а не очень очень близкие к нему приближения?
399291
#296 #399265
>>396075 (OP)
аноны, выручайте. Как привести (Cos(4x)-Cos^3(4x))/(3x^2) к первому замечательному пределу?
399298399342
#297 #399291
>>399259
Все эти пределы равны одному и тому же вещественному числу. Это устанавливается посредством тригонометрических функций - точнее, их рядов Тейлора.
#298 #399298
>>399265
Весь семестр пинал хуи, а теперь хочешь, чтобы тебе решали задачки?
В армию иди, пидор.
399319
#299 #399319
>>399298

>В армию иди


Может ему еще и Родину любить?
399326399331
#300 #399326
>>399319
Сам виноват, теперь Родина любит его.
#301 #399331
>>399319
Любить не обязательно, достаточно говфифьфя.
#302 #399342
>>399265
(Cos(4x)-Cos^3(4x))/(3x^2)
((Cos(4x)(1-Cos^2(4x))/(3x^2)
(Cos(4x)Sin^2(4x))/(3x^2)
Cos(4x)(16/3)(Sin(4x)/4x)^2
399908
#303 #399739
Бампну разок.
#304 #399756
Привести биекцию с (0; 1) на [0; 1]

Ноль в 0.1, 0.1 в 0.01, 0.01 в 0.001...

Один в 0.9, 0.9 в 0.99, 0.99 в 0.999...

Остальное в себя

Правильно всё?
399764400646
#305 #399764
>>399756
0.(9) = 1 though
399800
6 Кб, 424x380
#306 #399765
Поясните, что такое функционал. Функция понятно что, а чем она отличается от функционала? Гуглил - нихуя не понятно.
399773400433
#307 #399773
>>399765
Функционал = функция от функции.
Формально особо ничем не отличается, просто при рассмотрении какого-то пространства функций и пространства функций на нём нужно как-то отличать одно от другого.
399777
#308 #399777
>>399773
Но функция от функции - это ж суперпозиция, не? Н-р g(f(x)) - это разве функционал?
399917
#309 #399781
>>396075 (OP)
В теории автоматического управления кто нибудь понимает? Критерии устойчивости, лафчх и прочая хуйня. нужно построить лфчх а я даун в тригонометрии
399802
#310 #399800
>>399764
Но ведь его никогда не достигнет никто
400770
#311 #399802
>>399781
В чем вопрос то?
#312 #399908
>>399342
Чёт нихуя не пойму
#313 #399917
>>399777
Ты фи зик дохуя? Подумай про функцию как про отображение.
5 Кб, 450x167
#314 #399933
Как перейти от второй строчки к третьей (пикрел.)? Или хотя бы что гуглить, чтобы понять?
Тета и Y - векторы, X - матрица, если нужно больше пояснений, см. исходную статью https://habrahabr.ru/post/307004/ , а вообще это вывод формулы для линейной интерполяции.
Быстрый гуглеж по запросам "линейная алгебра производные" и "матрицы производные" то ли ничего не дал, то ли я туплю.
399950399982
#315 #399934
*А - матрица
#316 #399949
Алло, 2017 - простое число. Такие дела.
#318 #399982
>>399933
Чтобы найти решение моей проблемы, мне пришлось гуглить очень долго, но это таки удалось!
http://nabatchikov.com/blog/view/matrix_der
#319 #400000
Где можно почитать исследование двухмерных (а, может, даже и трёхмерных) кривых, задаваемых в форме [мгновенный_радиус_кривизны]([общая_длина_начиная_от_точки_порождения]) {ни координаты точки порождения, ни ориентация порождающего вектора частью функции не являются}? И чтобы, обязательно картинок с хитрожопыми графиками самых разных таким образом заданных кривых были, да побольше.
400019400022400168
48 Кб, 1205x92
#320 #400004
Ребят, помогите решить этот ад, пожалуйста
400017400048
#321 #400017
>>400004
Эмпирически получается a^(НОД(m,n))-1
Теперь надо понять, как это доказать.
400048
#322 #400019
>>400000
У тебя эпичный гет, но я не знаю, что тебе ответить...
#323 #400022
>>400000
Аналитическая геометрия? Дифференциальная?
#324 #400048
>>400004
Где ты берешь такие задачи?

По сабжу: я придумал, как доказать, что аkn-1 делится на an-1:
тупо считаем (an-1)(an(k-1)+an(k-2)+...+an+1)=ank-an(k-1)+an(k-1)-an(k-2)+...-an+an-1)=аkn-1 (все сокращается, кроме нужного)

Это доказывает, что и аn-1, и аm-1 делятся на аНОД(m,n)-1 (см. >>400017 ), но не доказывает, что не существует общего делителя побольше. Продолжаю думать.

Если я непонятно объясняю, переспрашивай.
400092
#325 #400092
>>400048
Спасибо. На матфаке, лол.

А вообще: http://math.stackexchange.com/questions/678730/gcd-number-theory-proof-an-1-am-1-am-n-1?noredirect=1&lq=1

Только последний шаг для меня остаётся загадочным
#326 #400168
>>400000
У какие запросы, губа не дура :3
Но тебе это не положено.
Такие дела.
#327 #400232
Почему в определении функции всегда оговаривается, что иксу/иксам соответствует единственное значение у? Почему это так принципиально? Что плохого в том, что их будет несколько? знаю, что есть многозначные функции, но их же тоже не просто так в отдельную петушарню убрали
#328 #400233
>>400232
Полная хуйня, куча многозначных и они совершенно обычны - те же дробные степени, например.
Так и комплексные будут чем-то удивительным, лолка.
#329 #400412
>>400232
Подчеркивают инъекцию.
y=x^2 в R, например, не является таковой.
#330 #400426
>>400232
Функция это просто отображение из одного множества в другое, никто не говорит про сюрьекцию/иньекцию
#331 #400429
>>400232
Просто с таким объектом удобно работать. Много доказаных теорем и так далее.
#332 #400433
>>399765
Функция - отображение произвольных множеств f: X -> Y, функционал - отображение произвольного множество во множество действительных числе f: X -> R
400451400471
#333 #400451
>>400433
Как раз под функцией чаще всего подразумевается отображение в вещественную прямую R или произвольное кольцо R, а X → Y, где X и Y — множества называется отображение.
400465400466
#335 #400466
>>400451
отображение это еще и подмножество множества X,Y
400467
#336 #400467
>>400466
XxY - декартова произведения
#337 #400471
>>400433
Функционал это отображение в любое поле, разве нет?
#338 #400561
>>400465
Такое идиотское определение присутствует исключительно в русскоязычной википедии. В английской функционал определяется как функция из векторного пространства (= пространства функций) в скаляры.
400563
#339 #400563
>>400561
"Математическая энциклопедия" Виноградова подтверждает приведенное в рувики определение, мотивируя Колмогоровым.
ПОПРОБУЙ ОПРОВЕРГНУТЬ #340 #400646
>>399756

ПРОТИП: ТЫ НЕ МОЖЕШЬ
#341 #400770
>>399800
А сумма ряда 1/х^2 никогда не достигнет двойки, но её все считают равной двум.
Улавливаешь?
400786400833400882
#342 #400786
>>400770
Ну тогда 1 в 0.8 в 0.88 в 0.888 и т.д. Годится?
400831
#343 #400831
>>400786
Выбираешь последовательность в (0,1) и делаешь биекцию между ней и ней же + точка 0 и точка 1.
Остальное на месте

Пример:
Пусть последовательность 1/q^n
n с 1 до бесконечности.
0 -> 1/q
1 -> 1/(q^2)
1/q -> 1/(q^3)
1/(q^2) -> 1/(q^4)
....
1/(q^k) -> 1/(q^(k+2))
401251
sage #344 #400833
>>400770

>но её все считают равной двум.


Кто все-то, ты тут один нахуй, поехавший.
#345 #400846
Двач, помоги доказать что НОД(ta, tb) = tНод(a, b)
Только не для целых чисел а для произвольного целостного кольца (не обязательно факториального). Вроде простая вещь а что-то застрял и туплю, не могу доказать.
401514
#346 #400882
>>400770

> сумма ряда 1/х^2 никогда не достигнет двойки, но её все считают равной двум


Чиво?
401257
#347 #401251
>>400831
А с моим что не так то?
401262
#348 #401257
>>400882
Он хочет сказать, что в последовательности частичных сумм этого ряда никогда не встретится число 2, но он не знает таких слов.
#349 #401262
>>401251
А 0 куда?
401380
#350 #401380
>>401262
Я ЖЕ СКАЗАЛ БЛЯДЬ 0 в 0.1 в 0.11 в 0.11 етц
100 Кб, 1075x903
#351 #401428
Наукач, помоги залетному. Есть одно правило Лопиталя на пике и я не понимаю доказательства случая для бесконечности. Чем оно отличается от случая 0/0?
401483401496
#352 #401477
>>397973

Гугли W - функцию.
#353 #401483
>>401428
Производная показывает наклон касательной, или что то же самое, скорость изменения значения функции.
Если у функции вверху/внизу дроби скорость ухода в бесконечность больше чем функции внизу/вверху дроби то получается какое то число.
Если же функция вверху/внизу дроби имеет очень большую производную, скорость изменения, по сравнению с второй функцией, то дробь уходит в бесконечность быстрее чем ты успеваешь сказать абрвалг и нужно применять правило клопиталя еще и еще
#354 #401496
>>401428
Смотри доказательство по базе. там все случаи сразу.
#355 #401514
>>400846
надо доказать исходя из дистрибутивности и существования (ta,tb)
#356 #401520
Господа, каким методом берётся интеграл от выражения
(4x^2 + 1) sqrt(x)/(2 x^2) exp(-x)
По частям не выходит, а курс матана уже подзабылся. Заменой тоже вроде так себе, короче, я немного в ахуе.
#357 #401572
>>397246
А когда язык стал формальной системой? Язык -- это единственный способ описания мира. Что же ещё математика? Ты считаешь, что законы математики существуют вне нашего разума, они материальны или имеют физический смысл? Математика по отношению к реальности -- это как отражение по отношению к отражаемому. Да что там, даже если твой аргумент будет не на столько идиотским, он всё-равно будет глупым. Почитай Витгенштейна, ЛФТ. Первые пару глав, и всё станет ясно само собой. Если увлечёшься, то и "философские исследования" стоит почитать.
А, кстати, что вообще язык, по-твоему?
#358 #401578
Математикобоги, не сочтите за дерзость, посоветуйте литературу для изучения Матери всех наук, если в школьные годы по глупости себя этим занятием не утруждал, а ныне, слезно раскаиваясь, хочу вновь взяться за ум. Знания практически отсутствуют, как в алгебре, так и в геометрии, но желание велико.
401580401582
#359 #401580
>>401578
Список книг в разделе /math https://2ch.hk/math/res/6879.html (М)
#360 #401582
>>401578
На первом же курсе приличного с точки зрения мат образования факультета поймёшь, что для настоящей математики понятия алгебры и геометрии слишком обширные, а в школе -- бредовые. Если совсем того, то лучше (реально лучше) Петерсона за 6 класс брать, потом каких-нибудь профильных школ 7-8-9 класс, потом решать олимпиадки всякие разные, ну а дальше можно заняться хотя бы сколько-нибудь серьёзной математикой, вроде Фихтенгольца по матанализу. Да и, кстати, выбор литературы предельно сильно зависит от конкретных целей. Если хочешь решать задачки, то по линейной алгебре будет лучше Беклемишев, если хочешь глубинного понимания, то ближе Костринкин, фор экзампл.
401589401590
#361 #401589
>>401582
Учусь в меде и хотелось бы в физику углубиться, т.к. мед науки во многом с физикой связаны. Плюс мозг в технтческом плане станет лучше работать (надеюсь), а это в изучении химии чрезвычайно способствует.
401618
#362 #401590
>>401582
Да и непосредственно физика интересует, что уж там говорить.
#363 #401618
>>401589
пиши мне, могу пояснить за физику и матан, даже вводный курс тебе провести
puphistX#|ickANUSgmaBZ1ilPUNCTUMc6c3om
#364 #401809
Ладно. Вы ведь тут очень умные и знающие математики? Кто-нибудь наверняка сверх необходимого, из интереса, изучает всякие там исследования, разработки и нерешённые задачи. Так?
Вот расскажите мне, есть ли такая штука, что благодаря некоторым силам какие-либо уравнения или системы уравнений могут или давать определенный результат, или продолжать действия, при том меняя свои переменные и преобразуя сами функции и формулы? Я не знаю как это описать и как это должно в полной мере выглядеть, но в среде определенных чисел такие системы должны принимать такие значения, что при различных условиях возможно было бы найти функции, описывающие зависимость значений. Но при этом эти находимые функции не находят любое решение, они определяют диапазон, необязательно последовательность, и сами имеют неизменное колличество действий.
Дополню что системы, что я описываю изначально, сами определяют диапазон, а исходные функции необходимо определть уже по корреляции значений чисел или выводов этого диапазона. Диапазоны могут быть различны, массив данных будет гораздо больше возможного для обработки.
401822
#365 #401822
>>401809
шизофазик в треде, скорее в машину
Кто у мамки молодец #366 #401846
На столе лежат две одинаковые монеты. Пусть одна из них лежит неподвижно, а другая обкатывается вокруг нее, все время с нею соприкасаясь. Сколько оборотов вокруг своей оси сделает вторая монета, обойдя один раз вокруг неподвижной монеты?
Примечание: подвижная монета все время находится в плоскости стола.
Сколько оборотов вокруг своей оси?
401848401856
#367 #401848
>>401846
Это физическая задача а не математическая. Да она и притом не абстрактная. Если хочешь, чтобы я её решил, тебе придётся составить уранение/систему уравнений с неизвестными.
401853
#368 #401853
>>401848
Узнай эмпирическии. Пробздись.
и охуей
401895
#369 #401856
>>401846
Очевидно, один.
401858
#370 #401858
>>401856
Чё, двух монет в кармане нет? Проверь.
401882
#371 #401882
>>401858
Блять. Хули два-то. Сука.
Хотя, очевидно. Если мы совместим монеты как шестерёнки и прокрутим на один оборот, то каждая сделает один оборот. А если мы перенесём систему отсчёта в одну из монет, то вторая займёт вертикальное положение дважды, только контактируя разными боками. Лол.
401888
#372 #401888
>>401882
Что ты несёшь?
Фильтр Калмана #373 #401889
Здарова матиматиканы, кароч надо раскурить фильтр калмана. Нужно для оценки величины параметра, завсящего косвенно от двух других. Кто что может посоветовать почитать, или пояснить?
#374 #401890
>>401889
Интересует сигма точечный фильтр калмана(Unscented Kalman filter)
#375 #401893
>>401889
Как вообще использовать матрицы ковариации? У меня всего две полученые величины. Как я могу попарные ковариации из них получить? И собрать их в матрицу?
#376 #401895
>>401853
Математика не занимается эмпирическими фактами. И не решает задачи эмпиречески.
401905
#377 #401905
>>401895
Общепризнанное мнение и то, что каждый считает давно решённым, чаще всего заслуживает исследований. (Г.К. Лихтенберг).

"Я верю, что числа и функции анализа не являются произвольными созданиями нашего разума: я думаю, что они существуют вне нас в силу той же необходимости, как и объекты реального мира, и мы их встречаем или их открываем и изучаем точно так, как это делают физики, химики и зоологи" (Ш. Эрмит)

НИНУЖНО (Аноним)
401910
#378 #401910
>>401905
Первое высказывание вообще не к месту. Что ты им хотел сказать? Что математики должны по десять раз решать тупые задачи, которые давно решены физиками? Ещё скажи, что все должны по десять раз трястись над одним и тем же интегралом, которые был взять двести лет назад и проверять эксперементально правильно ли он решен! В математике эмперический метод практически бесполезен, поскольку его можно провести только конечное число раз, а матерует и бесконечноми сущностями. Например, можно эмпирически пару раз проверить верна ли формула квадратных уравнений, но нельзя таким способом доказать её верность для всех чисел а значит и верность в общем случаи.

>Я верю


Ключевое слово - верю. Кто-то верит в христа, кто-то в аллаха, кто-то, что математика постоенна алгориметрически и аксиома выбора верна только для конечного числа чисел. У тут научная доска, а религиозная.

>что числа и функции анализа не являются произвольными созданиями нашего разума: я думаю, что они существуют вне нас в силу той же необходимости, как и объекты реального мира, и мы их встречаем или их открываем и изучаем точно так, как это делают физики, химики и зоологи


Филосовский вопрос, который не интересен математикам, поскольку ответ на него по-факту ничего не меняет. Физическа построенна на эксперементах, математика нет. Даже если они что-то и пытаются это искать, то после того как докажут, это становится незыблемым камнем, который не нужно больше проверять, с физикой дела не так.
401913401915
#379 #401913
>>401910

>НИНУЖНО


Ясно.
401917
#380 #401915
>>401910

>постоенна алгориметрически и аксиома выбора верна только для конечного числа чисел. У тут научная доска, а религиозная.


Вынь хуй из рота и реши, наконец, эту задачу.
401917
#381 #401917
>>401913
Что именно не нужно? Решение физических задач математикам? Не нужно. Ты прав.
>>401915
Я не пизик, иди нахуй. По твоему Бурбаки и Гротендик стал бы опускаться до такого низкого уровня? Нет.
401918401919401921
#382 #401918
>>401917
*физик
#383 #401919
>>401917

>Бурбаки и Гротендик


Два дебила - это сила.
401920
#384 #401920
>>401919
Но ведь дебилы могут быть только физиками, вот Эйнштейн переоценённый даун.
401927
#385 #401921
>>401917
Вот тебе условия, вот тебе данные - реши задачу,бро?
@
Я не пизик, азаза
401922
#386 #401922
>>401921
Это пизические условния, мне же нужны математические. Смотри, из этой задачи ты должен вычленить математическую часть, переписать в виде уравнений и тогда я её решу.
401926401927
#387 #401926
>>401922
Оk педант, сколько оборотов вокруг своей оси сделает монета, достаточно абстрактно, или математики не могут это представлять?
401928
#388 #401927
>>401920
>>401922
Угомонись, неполноценный.
Это не троллинг - ты просто обмазываешься говном публично, не более.
401928
#389 #401928
>>401926
Это всё ещё физика. Дай определения оси, оборота, момента.
>>401927
А вот и пизик порвался.
401930401931
#390 #401929
>>401889
бамп вопросу
#391 #401930
>>401928
define определение
401981
#392 #401931
>>401928

>Дай определения оси, оборота, момента.


Пойдут общепринятые.
401981
#393 #401946
>>401889
ну бамб же, никто не подскажет хоть что то?
401971
#394 #401957
>>401889
Бамп вопросу.
401971
#395 #401966
>>401889
Бамп же, чот у анона математику не очень идея походу.
401971
#396 #401971
>>401946
>>401957
>>401966
Соси хуй, быдло.
#397 #401981
>>401930
Филосовский вопрос, иди с таким в /ph.
>>401931
Теперь опиши задучу в этих терминах. С заданной системой уравнений.
>>401889
Вторая культура.
#398 #402007
>>396075 (OP)
Есть какие то современные книги по вышмату для прикладного применения вышмата? Тобишь без теории множеств, топологий, аффинных преобразований на двадцатой странице учебника и т.д.
122 Кб, 719x960
#399 #402117
Привет /sci/, наткнулся на небольшую проблему, в принципе на листочке все казалось бы просто, бери дифференцируй да подгоняй ответ под нужное выражение, без особых проблем решил 15\14, но вот на 13 я застрял, продифференцировал, получил sin(x), но чего-то не могут понять как из косинуса в данном случае получить синус, пробовал через тригонометрическое тождество, но там получается сложение, а в исходном выражении сложения нету. Не очень хочется в биореактор, знаю что мне уже пора туда, но все же хотелось бы разобраться.
402118
#400 #402118
>>402117
ну к примеру a=2, t=1 b=-1 c = pi/2 дает тождество.
402119402120
#401 #402119
>>402118
a = 1
быстрофикс
#402 #402120
>>402118
Точно, формулы приведения! Спасибо анон, может я еще чуть позже отпишусь, если дальше тоже не получится с другими примерами.
#403 #402124
Двач, поясни мне пожалуйста нахуй нужен Ряд Фурье?
402143
#404 #402143
>>402124
Чтобы мп3 играть.
#405 #402163
БАМП
53 Кб, 640x480
#406 #402185
Привет!
Подскажите, пожалуйста, программу, в которой можно "от руки" нарисовать график, а программа выдаст массив значений.
402188
#407 #402188
>>402185

>4367935+de8accb[...].jpg


лишь неделю назад понял, что такое большое колесо на старинных велосипедах было вызвано необходимостью убрать у лошади желание перепрыгнуть эту конструкцию ударив ездока копытом по ушам
402189
sage #408 #402189
>>402188
А нахуя лошади перепрыгивать велосипед?
Велосипед-то с педелями на переднем колесе, так что я думаю, что размер колеса связан с оптимальным соотношением число оборотов/скорость.
Ладно, блядь, с оцифровкой графиков разобрался, теперь скажите мне, как имея набор значений функции найти саму функцию.
402196
#409 #402196
>>402189
Открытая математическая проблемма.
Метод наименьших квадратов
1 Кб, 231x31
#410 #402202
Продублирую из /math/, только хуй знает зачем.

У меня там вроде бы как квадратное тригонометрическое уравнение с параметром, но я, решая это уравнение, каждый раз получаю новый ответ. В общем, я уже запутался и не знаю, что со мной не так.

Я не прошу решить это задание, я прошу натолкнуть меня на верный ход мыслей.

Условие такое: при каких p уравнение имеет корни?
402206
#411 #402206
>>402202
ну если квадратные уравнения умеешь с параметром решать, то думаю проблем возникнуть не должно.
402208
#412 #402208
>>402206
Я делаю замену синуса на t, решаю уравнение, дохожу до дискриминанта, мне должно хватать условия, что если D>=0,то корни есть. Это ведь ты имеешь ввиду?
402209
#413 #402209
>>402208
Хотя, стоп. Если D>=0,то у меня есть корни относительно уравнения с t, но не для первоначального уравнения. Получается, D>=0 — это одно из условий?
402210
#414 #402210
>>402209
Осталось вспомнить, что синус функция ограниченная, вопрос к тебе: чем она ограничена?
402211
#415 #402211
>>402210
От -1 до 1 строго.
402213
#416 #402212
Я должен составить систему из условий. Первое условие — это для D. Второе — оганичения синуса. Что-то еще?
#417 #402213
>>402211
почему строго? разве синус не может равняться единице?

>Что-то еще?


этого должно хватить
402214
#418 #402214
>>402213
Тьфу. Включая единицу. А если решать графическим способом, то не нужно рассматривать случаи, когда один из корней лежит за пределами промежутка от -1 до 1?
402216
#419 #402216
>>402214
Если корень т.е sin(x) по модулю оказался больше единицы, то нет смысла его рассматривать, потому что уравнение sin(x) = t где |t|>1 не имеет решения в вещественных числах.
402219
#420 #402219
>>402216
Благодарю.
#421 #402334
Операция умножения, есть ли она в реальном материальном проявлении? Или это абстракция в математической модели?
Ну например сложение естественно:
Есть тут 3 яблока, а там 5, ты их скинул в кучу, пересчитал и получил 8. Вот иллюстрация сложения.
А как показать умножение с яблоками? Его же нету...
402340402344402524
#422 #402340
>>402334
Потому что умножение это соркащенная запись опредленного числа суммирования одинаковых чисел.
Чтобы не писать +5 яблок+5 яблок+5 яблок+5 яблок+5 яблок+5 яблок+5 яблок+5 яблок+5 яблок+5 яблок+5 яблок+5 яблок+5 яблок+5 яблок+5 яблок+5 яблок+5 яблок+5 яблок+5 яблок+5 яблок+5 яблок+5 яблок+5 яблок+5 яблок+5 яблок+5 яблок+5 яблок+5 яблок+5 яблок+5 яблок+5 яблок+5 яблок+5 яблок просто пишут 5 яблок*33
#423 #402344
>>402334
3*5 это 3 ящика с 5 яблоками, так понятно?
402345
#424 #402345
>>402344
Да я про другое спрашивал. Забавно что абстракция начинается уже с умножения. А это там 2-ой класс..
402364
#425 #402364
>>402345
Какая нахуй абстракция? Просто сокращенная запись суммирования.
Открой учебник первого класса где то в конце и там прямо про это писать будут
402369
#426 #402369
>>402364
Ну так это сокращение само по себе не существует.
Нельзя обучить умножение до этого не обучив счету и сложению.
402370
#427 #402370
>>402369
Т.е это операция уже внутри мат модели, как там возведение в степень - тоже сокращение.
#428 #402443
>>396140
Вот мой тренд, который не взлетел. Я планировал именно такими книжками и делиться. >>398226 (OP)
#429 #402524
>>402334
сложение это тоже абстракция.
Что такое восемь яблок? Это 11111111 яблок, обозначенные странным значком. А если мы будем складывать допустим лис и зайцев, то получится, что какое-то количество зайцев съедят и будет другое сложение. А если в нашей корзине дырки, то бумага + ягода и ягода + бумага будут давать разный результат. Все зависит от модели в которой мы работаем.
402543
#430 #402543
>>402524
Такой подход фейлится при определении вещественных чисел. Под капотом каждого вещественного числа скрывается бесконечность, поэтому вещественное число нельзя рассматривать как абстракцию от яблок - в реальном мире бесконечностей нет.
402583402758
#431 #402583
>>402543
Как раз таки в реальном мире одни бесконечности, а вот яблок - нет.
#432 #402651

>И только математика по настоящему абстрактна и не привязана к материальному миру, благодаря чему можно постичь непостижимое



Гуманитарий детектед. Математика привязана к особенностя человечекого сознания и мозга - эта реально субъективная методологическая фигня основанная на том как моск может думать. При этом реальный мир плохо ложиться в прокурстово ложе математики - уже сейчас очевидно что с помощью математики невозможно точно описывать объективную реальность, но моск сука по другому думать не умеет и наука топчется из-за этого на месте. В общем нашел на что дрочить.
402668402670
#433 #402668
>>402651

>реальный мир плохо ложиться в прокурстово ложе математики - уже сейчас очевидно что с помощью математики невозможно точно описывать объективную реальность


Развивай пруфы
#434 #402670
>>402651

>эта реально субъективная методологическая фигня основанная на том как моск может думать. При этом реальный мир плохо ложиться в прокурстово ложе математики - уже сейчас очевидно что с помощью математики невозможно точно описывать объективную реальность



Гуманитарий детектед, который никогда не видел Нётеровых зарядов, группы Пуанкаре, симметрий в Стандартной Модели и проч.
402751
#435 #402751
>>402670
А ты сам-то видел?
#436 #402758
>>402543
если мы конструктивно определим вещественные числа как последовательности и будем считать только до определенного знака, то такой проблемы нет.
402760402822
#437 #402760
>>402758
При таком подходе понятие равенства двух вещественных чисел, вообще говоря, потеряет смысл.
402821402989
#438 #402819
Надо """"""""""""""""""""""проверить"""""""""""""""""""""""", что для рациональных чисел выполняются все аксиомы действительных помимо непрерывности.

Что от меня хотят, блядь!?!
#439 #402821
>>402760
Нет, с чего это? Ты же можешь сравнивать последовательности.
402837
#440 #402822
>>402758
Они так и определены блядь аутист. Книжку бы в руки взял, а не хуй и флешлайт.
402989
#441 #402837
>>402821
0,(9) = 1?
402874
#442 #402874
>>402837
Нет. 0,(9) будет бесконечно близко к 1, но не равно ей.
402883402885
#443 #402883
>>402874
Нет, это джве разных записи одного числа.
#444 #402885
>>402874
Понятно. Идите с таким определением нахуй.
#445 #402899
>>396843
Как это ты умножил ноль на 4 и получил 28? Научи
9 Кб, 399x105
#446 #402904
Что за перпендикулярные маленькие отрезки изображены на треугольнике?
402953
#447 #402936
>>397714
А тут что только женщины сидят?Я не хочу никого унизить просто я спрашиваю.
403059
98 Кб, 705x359
14 Кб, 664x89
#448 #402949
3. p=4
4. 2-й пик
7. 4 точно не является, думаю также, что 3 и 5 также не являются
8. 5-й порядок
Правильно ли я думаю?
#449 #402953
>>402904
это "черточки" для обозначения равных отрезков. т.е нижняя сторона треугольника разделена на два равных отрезка "две черты". Боковые стороны равны "одна черта"
#450 #402989
>>402822
нихуя они так не определены, у них мощность континуума.

>>402760
равенство до определенного знака опять же.
402996
#451 #402996
>>402989
0.(9) и 1 равны как вещественные числа, но не равны как последовательности.
45 Кб, 450x554
#452 #403022
Поясните гуманитарию на примерах, что такое
- отрицательная вероятность
- трансфинитные числа

Читал Вики. Ни хуя не понял.
403033
9 Кб, 356x236
#453 #403024
Дубль из /math/
Нужно посчитать площадь проекции трёхосного эллипсоида размерами a,b,c на произвольную плоскость Ax+By+Cz=0.
Пытался влоб — не осилил. Ответ вроде интуитивно прост, но не всплывает. Я так понял надо аффинные преобразования курить, чего стоить читануть по этому поводу?
403061
#454 #403033
>>403022
1. Отрицательная вероятность - крайне нестандартное понятие. Вероятность, согласно аксиомам Колмогорова, всегда неотрицательна. В учебниках отрицательная вероятность не встречается.

2. Натуральные числа - это 0, 1, 2, 3, 4, 5, ... и так далее. Самого большого натурального числа нет. Однако за всеми натуральными числами идёт первое трансфинитное число ω0. С него начинается как бы новый натуральный ряд. ω0+1, ω0+2, ω0+3, ... и так далее. Потом идёт число ω00, за ним ω00+1, ω00+2, ω00+3, ... и так далее - ещё один натуральный ряд. Этот процесс продолжается неограниченно. За далью даль, за каждым рядом - ещё один такой же ряд.

Дело в том, что каждое число - это множество всех предыдущих чисел. 0 - это пустое множество, 1 - это множество {0}, 2 - это множество {0,1}, 3 - это множество {0,1,2}, и т.д. Так вот, ω0 - это множество всех натуральных чисел, ω0+1 - это множество, содержащее все натуральные числа а также число ω0. А число ω00 - это множество, содержащее все натуральные числа и все числа вида ω0+n, n - натуральное.

Числа, о которых я говорю, называются "ординалы". Натуральные числа - конечные ординалы, все остальные ординалы - бесконечные. Ординалы, у которых есть предшественники, называются непредельными. Остальные называются предельными. ω-числа - типичные предельные ординалы. Каждый предельный ординал есть не более чем множество предыдущих ординалов.

Каждый ординал - это множество. У множеств есть мощность, поэтому у ординалов она тоже есть. ω0 - счетный ординал, ω00 и другие подобные тоже являются счетными. Первый несчетный ординал называется ω1, все последующие ординалы также несчетны. Для каждого множества ординалов существует ещё больший ординал. Самого большого ординала нет.

Формально ординалы определяются с помощью конструкции фон Неймана. Её я не приводил. Сказанное выше - просто пояснения.
#455 #403059
>>402936

:Жалкая попытка, лентяйка, тебе никто ничего не должен.
#456 #403061
>>403024
Хуиные преобразования. Приводишь к системе координат (A,B,C) и два рандомных ортогональных этому направлению вектора, далее, к каноническому виду, и считаешь площадь (эллипса).
22 Кб, 627x630
#457 #403123
Возможно спрашиваю не по адресу, но не знаю где ещё. Как в matlab найти абсциссу точки пересечения кривых?
Ну напимер у нас есть y1=ln(x) и y2=x^3. Как найти X при котором y1=y2? Я знаю что надо приравнять к нулю, но как в матлабе?
403136
309 Кб, 2388x932
#459 #403171
Антош помоги с матаном
403176
#460 #403176
>>403171
Нет. /sci/ не помогает с домашками, это принципиальное правило.
#461 #403183
>>403136
Ой, спасибо. Я уже разобрался :^)
13 Кб, 300x269
#462 #403281
Поясните. Как известно, алеф-нуль - это кардинальное число множества N. Правильно ли, что алеф-нуль это натуральное число?
403284403294
#463 #403283
>>396075 (OP)
Посоветуйте как можно более полный учебник по математике ориентированный на даунов с тремя классами церковно-приходской школы. Чтобы всё разжевывалось максимально интересно и понятно, без воды, с интересными примерами где это можно применить и главное чтоб присутствовала история, как, когда и зачем такой-то математик открыл или вывел свою хуету.
#464 #403284
>>403281

> алеф-нуль это натуральное число


Каждое натуральное число число следует за каким-то натуральным числом (кроме числа 1 которое ни за чем не следует). За каким натуральным числом следует алеф-нуль?
403285
2433 Кб, Webm
#465 #403285
>>403284

>За каким натуральным числом следует алеф-нуль?


Алеф-нуль минус 1))) Не, если серьезно, мне правда непонятно как так. Есть множество натуральных чисел. Есть его кардинальное число. Но оно почему-то само по себе не натуральное число.
403288
#466 #403288
>>403285
Ну смотри, если б алеф0 было натуральным, натуральные числа можно было бы отобразить в некоторое конечное подмножество, а это, очевидно, не так.
#467 #403294
>>403281
Множество называется транзитивным, если каждый его элемент является его подмножеством.

Ординал - транзитивное множество, вполне упорядоченное отношением "x является элементом y".

Теорема 1.
Элементы ординала - ординалы.
Для каждого ординала x определен ординал x+1.
Каждый ординал содержит все предшествующие ординалы.
Пруф. В любой книжке по теории множеств.

Кардинал - ординал, не равномощный предшествующему ординалу.

Омега-нуль определяется как наименьший ординал, который вместе с каждым элементом x содержат x+1.

Два множества называются равномощными, если между ними есть биекция. Если первое множество равномощно второму, а второе - третьему, то первое и третье равномощны, очевидно.

Мощность множества ординалов определяется как наименьший кардинал, которому оно равномощно.

Алеф-нуль определяется как мощность множества омега-нуль.

Множество называется счетным, если оно равномощно множеству натуральных чисел.

Множество называется конечным, если оно не содержит счетного подмножества.

Натуральными числами называются мощности конечных множеств.

Теорема 2. Мощность множества натуральных чисел есть алеф-нуль.
Пруф. Множество натуральных чисел вместе с каждым числом n содержит число n+1, значит, между ним и омега-нуль есть биекция. По т.1, мощность омеги-нуль - алеф-нуль.

Теорема 3. Алеф-нуль не является натуральным числом.
Пруф. Алеф-нуль равномощен множеству натуральных чисел. Значит, он не является конечным множеством. Значит, он не является натуральным числом - они все конечны по определению.
#467 #403294
>>403281
Множество называется транзитивным, если каждый его элемент является его подмножеством.

Ординал - транзитивное множество, вполне упорядоченное отношением "x является элементом y".

Теорема 1.
Элементы ординала - ординалы.
Для каждого ординала x определен ординал x+1.
Каждый ординал содержит все предшествующие ординалы.
Пруф. В любой книжке по теории множеств.

Кардинал - ординал, не равномощный предшествующему ординалу.

Омега-нуль определяется как наименьший ординал, который вместе с каждым элементом x содержат x+1.

Два множества называются равномощными, если между ними есть биекция. Если первое множество равномощно второму, а второе - третьему, то первое и третье равномощны, очевидно.

Мощность множества ординалов определяется как наименьший кардинал, которому оно равномощно.

Алеф-нуль определяется как мощность множества омега-нуль.

Множество называется счетным, если оно равномощно множеству натуральных чисел.

Множество называется конечным, если оно не содержит счетного подмножества.

Натуральными числами называются мощности конечных множеств.

Теорема 2. Мощность множества натуральных чисел есть алеф-нуль.
Пруф. Множество натуральных чисел вместе с каждым числом n содержит число n+1, значит, между ним и омега-нуль есть биекция. По т.1, мощность омеги-нуль - алеф-нуль.

Теорема 3. Алеф-нуль не является натуральным числом.
Пруф. Алеф-нуль равномощен множеству натуральных чисел. Значит, он не является конечным множеством. Значит, он не является натуральным числом - они все конечны по определению.
403295403296
#468 #403295
>>403294

>По т.1, мощность омеги-нуль - алеф-нуль.


по определению
#469 #403296
>>403294

>Множество называется транзитивным, если каждый его элемент является его подмножеством.


>Ординал - транзитивное множество, вполне упорядоченное отношением "x является элементом y".


>Кардинал - ординал, не равномощный предшествующему ординалу.


Все это справедливо для алеф0 же.
403314403367
#470 #403314
>>403296
Да. И?
403364
31 Кб, 300x300
#471 #403317
Наверное не совсем правильно писать сюда, но все же.

Я недавно начал работать в ресторане, у нас там около 100 столов, каждый работник ответственнен за примерно 20, на каждом столе салфетница, в ней 12 салфеток, и дело все в том, что с утра когда никого нету мы в основном приготавливаем резервные салфетки для салфетниц которые в течении дня пустеют, меня это заебало и я понаблюдал за их расходом.
Получается, что чем больше салфеток в салфетнице, тем менее экономнее их тратят, например когда их остается 2-4 они лежат часами так же как пачка из 12 спускается с 12 до примерно 5. И вот я начал класть по 8 салфеток в салфетницу и по времени вроде как выходит одно и тоже, но там была контрольная группа из 2 столов, больше не смог ибо в часы пик бегаю как угорелый.

Сам я человек в точных науках очень отсталый, в школе было стабильно 2 по математике с 3 по 11 класс, до 7 класса не понимал, что такое дроби, но при этом как то сдал ЕГЭ лол Как бы все это правильно рассчитать, хочу показать научную работу манагеру, с графиками там, хотя вангую, что она разабьется об какой нибудь маркетинг в духе у нах бохатое заведение. Но может повезет и придется меньше дрочиться с салфетками...

Короче, что и как правильно делать? наверное ещё смогу запросить на складе накладную с общей статистикой по ежемесячным закупкам салфеток
403322403350403376
#472 #403322
>>403317

Или это больше социологическая задача? я вот не ебу на самом деле.
403350
246 Кб, 2933x1605
#473 #403329
Поясните пруф Кантора-Кокоши. Понятно, что a_m меньше или равно b_n. Но какого хуя с будет между a_n и b_n, и как из этого следует, что оно всем множествам последовательности принадлежит?
403350
#474 #403350
>>403329
У тебя последовательность вложенных интервалов. Интуитивно, левые концы будут потихоньку возрастать, а правые убывать. Множество 'А' есть ничто иное, как последовательность левых концов. Любой левый конец меньше любого правого. Другими словами, множество 'А' ограничено. По аксиоме/теореме о полноте, существует супремум 'А'. Обозначим его за 'с'. Это число
1) больше an. Это ясно, т. к. это верхняя гиань по построегию; Интуитивно, к нему стягиваются an.
2) меньше bm. Конечно, мы можем взять новое 'с' такое, что оно будет за пределами bn, и которое будет верхней гранью 'А'. Но супремум - наименьшая такая грань! Значит, оно меньше любого из правых концов.

Раз у нас число 'с' больше любого левого конца и меньше любого правого, то оно принадлежит всем интервалам.
Под иеньше/больше я везде понимал меньше или равно и т. д.

>>403322
>>403317
Первое. Если тебя интересует математическая часть, то нужно провести эксперимент, случайно выбирая столики и т. д. Скажем, измерить количество салфеток в первом случае. Потом заново, и еще, и еще. Отсюда полу, ишь среднее количнство оставшихся сплфеток. Потом ты изменяешь условия (кладешь меньше ижначально), и сновп несколько раз измеияешь, вычисляешб среднее.
После этого классическая мат. статистика (ANOVA/дисперсионный анализ, тут - тест Стьюдента) говорит тебе: со значимостью в 95%/99%/а% разницы между этими средними нет. Другими словами, количество оставшихся салфеток одинаково как при 12 начальных, так и 8 начальных.

Есди у тебя с математикой неоч, то вот другой вариант. Эффект, о котором ты говоришь, действительно существует - это scarcity heuristic. Не знаю, как в русской психологии переводят - вероятно, эвристика дефицита. Чем меньше ресурса остается, тем больше человек начинает его ценить. Заходишь на Google Scholar и вбиваешь там scarcity heuristic, или открываешь учебники по психологии, там должно быть написано. Куча статей есть на эту тему.
403354403393403399
#475 #403354
>>403350

>Интуитивно



Удивился (мимо залетный гуманитарий)
#476 #403364
>>403314
Ну значит по индукции алеф0 должен быть натуральным числом? Если он строится итеративно начиная с нуля? 0,1,2,3,...алеф0-1, алеф0.
403366
#477 #403366
>>403364

> алеф0 - 1


Ты не можешь использовать число "алеф0 - 1" пока не определишь операцию вычитания единицы для алеф0 либо не докажешь что алеф0, к примеру, натуральное (так как для натуральных числе операция вычитания уже определена). Естественно, что при доказательстве натуральности алеф0 нельзя использовать операцию вычитания (так как для ее применения нужно доказать что алеф0 натуральное). Насколько я помню, ты остановился на попытке указать натуральное число которое предшествует алеф0.
403367
#478 #403367
>>403366

>Ты не можешь использовать число "алеф0 - 1" пока не определишь операцию вычитания единицы для алеф0


Ну хуй знает. Если так: >>403296

>Множество называется транзитивным, если каждый его элемент является его подмножеством.


Очевидно, что единица является подмножеством алеф0, как и любое другое натуральное число. В итоге получится предшествующий ординал, т.к. алеф0 - кардинал.

>Кардинал - ординал, не равномощный предшествующему ординалу.

403375
#479 #403375
>>403367
Стань чуть тоньше, твои вопросы становятся менее интересными.

Для любого ординала определена операция inc.
inc X - это наименьший из ординалов, больших X.

Существуют предельные ординалы. Ординал x называется предельным, если нет такого ординала a, что inc a = x.

Теорема. Каждый алеф - предельный ординал.
#480 #403376
>>403317
Охуенный вопрос, кстати. Серьезно.
#481 #403393
>>403350

>Эффект, о котором ты говоришь, действительно существует - это scarcity heuristic. Не знаю, как в русской психологии переводят - вероятно, эвристика дефицита. Чем меньше ресурса остается, тем больше человек начинает его ценить. Заходишь на Google Scholar и вбиваешь там scarcity heuristic, или открываешь учебники по психологии, там должно быть написано. Куча статей есть на эту тему.



Я что та такое и думал, но не мог правильно выгуглить, сейчас буду дрочить эти исследования.
131 Кб, 1080x810
132 Кб, 1080x810
137 Кб, 1080x810
#482 #403398
Матаны, посмотрите, пожалуйста, кто-нибудь сможет сказать, какая это книга, автор? Мб кто-то из вас решал. Спасибо
403447
7 Кб, 1324x426
#483 #403399
>>403350
Я так понял, что ан и бм это фактически константы, а вот ам и бн могут двигаться как угодно между ними. Получается что с это ан, так как в таком случае оно подойдет для всех значений ам и бн включая предельный случай когда ам = ан = бн.

Но в таком случае достаточно взять любой непредельный случай и следующий интервал с ао > ан и тогда с уже не попадет в этот интервал ао бо.
403439
#484 #403439
>>403399
Ты только что супремум.
403534
#485 #403447
>>403398
Хз. Из всех известных мне задачников на "Группа А, группа Б" делил разве что Сканави.
#486 #403534
>>403439
Объясни нормально, мемзнаток.
#487 #403643
Ребят, кто понятным языком объяснит задачи по геометрии, (они типа легкие, но я не врубаюсь). 9класс, не бейте
#488 #403737
Посоветуйте хорошую литературу по теории графов.
#489 #403748
Есть уравнение (1). Есть уравнение (2), являющееся частным случаем (1). Есть утверждение A, делающееся относительно этих уравнений (например, относительно их решений).

(1) <= (2)

Правильно?

A(1) => A(2)

Правильно?
#490 #403783
>>396075 (OP)
Рисовал человечка, решил, что если начать с мужика, то будет неполиткорректно. Но если всех баб нарисовать, то будут думать, что ты поехавший сжв. Но нарисовав мужика понял, что математик в конце будет тоже мужиком, и будут все думать, что ты думаешь, будто бабы не могут в математику - нарисовал два мужика подряд.
336 Кб, 784x744
427 Кб, 764x767
#491 #403851
>>396075 (OP)
Привет математикам! Прошу пожалуйста отнестись с пониманием без рофлов.
Дело в том, что я год назад закончил нептуаколледж, что какбэ намекает.
Так вот, я хочу по крайней мере попытаться вырваться из стадий деградации и поступить на заочку в какой-нибудь ВУЗ, потому как ходить с корочкой ПТУ для меня сплошной стыд и срам. Для поступления надумал сдавать вместо ЕГЭ вступительные. Желанием ужасно горю, а вот мозгов - нет.
Знаю, что просьба моя возможно будет очень жирная, но может есть на примете какие-нибудь лекции желательно по темам на пике с разжевыванием, который поймет даже аутист?
Да, и именно видео. По книжкам мне как долбоебу нихуя не ясно.
Буду благодарен вам за вашу поддержку и понимание.
403865403912404003
#493 #403870
>>403865
Какой-то он ебанутый немного в сторону судьбы и Бога.
403871
#494 #403871
>>403870
Да. Но менее ебанутые занимаются наукой, а не пишут видео для школьников. Этот хотя бы компетентный.
#495 #403912
>>403851
Кхан академи, курсера.
44 Кб, 466x168
#496 #403926
>>403865
Серьезно?
403933403951404024
156 Кб, 808x1200
180 Кб, 761x1200
190 Кб, 923x726
#497 #403933
>>403926
Серьёзно. Просто не обращай внимания на такие заскоки.
#499 #404003
>>403851
попробуй еще зуланке-онищик
#500 #404024
>>403926
Насколько я знаю, что касается физики, он вообще первостепенный фрик, так что я бы опасался читать его впринципе.
404038
#501 #404038
>>404024
Книжка по топологии - вполне вин.
#502 #404387
Анон, как там подивает твой Бегемотик?
404402404416
#503 #404402
>>404387
Все бегемотики ускакали в /матх.
295 Кб, 940x655
#504 #404416
>>404387
Норм так, ничё.
#505 #404833
>>396124
>>396126
https://2ch.hk/sci/arch/ (М) тут возможно есть.
#506 #404987
Ребят спасайте это ад для меня
Двач помогай я нифига не понимаю!
Пик рандом
На двойном тетрадном листе бумаги (обяза-
тельно в клеточку) отметьте оси координат.
Отметьте на осях минимальные и максималь-
ные значения следующим образом. По оси Х
отсчет начните с нуля, поместив его в начало
шкалы. Расстояние в одну клеточку будет рав-
но единице (должно получиться по меньшей
мере 35 делений). По оси Y отступите от оси Х
одну клеточку и начните отсчет с числа, рав-
ного дню Вашего рождения. Например, если
Вы родились 14 мая, начните отсчет по оси Y с
14. Расстояние в одну клеточку также будет
равно единице (получится по меньшей мере 20
клеточек вверх).
В правом верхнем углу графика в узел сетки
(пересечение вертикальных и горизонтальных
линий сетки) поставьте точку (она должна
иметь координаты, которые возможно отме-
56
тить на графике). В левой нижней части графи-
ка, отступив от оси Х 3-5 клеток, а от оси Y –
только одну, также в узел сетки поставьте точ-
ку. Соедините эти две точки тонкой линией
(обязательно начертить карандашом и без ли-
нейки, от руки! Линия не должна быть идеаль-
но ровной — будет лучше, если она будет по-
чти прямой).
Теперь вдоль полученной линии, но не на
ней самой, отметьте в узлах сетки (на пересе-
чениях вертикальных и горизонтальных линий)
от 35 до 45 точек на удалении от неё не более
двух клеток. Постарайтесь расставить точки
таким образом, чтобы они не образовывали ко-
ридор, в центре которого будет проходить ис-
ходная линия. В идеальном варианте точки, на-
ходящиеся ближе к линии, должны встречать-
ся чаще, чем точки, расположенные от неё
дальше. Запишите целочисленные координаты
каждой точки в таблицу (она будет содержать
по меньшей мере 15 столбиков — рекомендуем
57
сначала определять, какие значения будут на-
ходиться в столбике, и только после этого отво-
дить под него место на листе.
Пример таблицы:
x y
1 15
2 17
2 16
... ...
Примечание. В таблице должны быть записа-
ны координаты всех отмеченных на графике
точек.
Посмотрите внимательно на график. Судя
по всему, он отражает некоторую линейную за-
висимость между показателями x и y. При этом
видно, что существуют погрешности измере-
ния, вызванные случайными факторами (а так-
же тем фактом, что измерения значений про-
водились с округлением до целых). Эти по-
грешности несколько искажают общую зависи-
мость, однако в целом вступает в силу закон
58
больших чисел, и искажения в общей массе
сглаживаются. Предположим, что отмеченные
точки выражают реальную ситуацию, которую
необходимо смоделировать.
Дальнейшие вычисления нужно проводить
вручную, поскольку это позволяет задуматься
над тем, что именно и правильно ли Вы делае-
те. Можно порекомендовать проверить полу-
ченные результаты расчетов на компьютере.
Самую трудоёмкую часть работы — построение
доверительных интервалов — разрешается де-
лать сразу на компьютере.
По полученным точкам постройте линейную
модель парной регрессии (y зависит от x) в
полном соответствии с рассмотренным в этой
лекции алгоритмом. Каждый пункт алгоритма
должен сопровождаться выводами по проде-
ланной работе и полученному результату, что
поможет Вам в дальнейшем при разработке об-
щих выводов и рекомендаций по практическо-
му использованию модели.
#506 #404987
Ребят спасайте это ад для меня
Двач помогай я нифига не понимаю!
Пик рандом
На двойном тетрадном листе бумаги (обяза-
тельно в клеточку) отметьте оси координат.
Отметьте на осях минимальные и максималь-
ные значения следующим образом. По оси Х
отсчет начните с нуля, поместив его в начало
шкалы. Расстояние в одну клеточку будет рав-
но единице (должно получиться по меньшей
мере 35 делений). По оси Y отступите от оси Х
одну клеточку и начните отсчет с числа, рав-
ного дню Вашего рождения. Например, если
Вы родились 14 мая, начните отсчет по оси Y с
14. Расстояние в одну клеточку также будет
равно единице (получится по меньшей мере 20
клеточек вверх).
В правом верхнем углу графика в узел сетки
(пересечение вертикальных и горизонтальных
линий сетки) поставьте точку (она должна
иметь координаты, которые возможно отме-
56
тить на графике). В левой нижней части графи-
ка, отступив от оси Х 3-5 клеток, а от оси Y –
только одну, также в узел сетки поставьте точ-
ку. Соедините эти две точки тонкой линией
(обязательно начертить карандашом и без ли-
нейки, от руки! Линия не должна быть идеаль-
но ровной — будет лучше, если она будет по-
чти прямой).
Теперь вдоль полученной линии, но не на
ней самой, отметьте в узлах сетки (на пересе-
чениях вертикальных и горизонтальных линий)
от 35 до 45 точек на удалении от неё не более
двух клеток. Постарайтесь расставить точки
таким образом, чтобы они не образовывали ко-
ридор, в центре которого будет проходить ис-
ходная линия. В идеальном варианте точки, на-
ходящиеся ближе к линии, должны встречать-
ся чаще, чем точки, расположенные от неё
дальше. Запишите целочисленные координаты
каждой точки в таблицу (она будет содержать
по меньшей мере 15 столбиков — рекомендуем
57
сначала определять, какие значения будут на-
ходиться в столбике, и только после этого отво-
дить под него место на листе.
Пример таблицы:
x y
1 15
2 17
2 16
... ...
Примечание. В таблице должны быть записа-
ны координаты всех отмеченных на графике
точек.
Посмотрите внимательно на график. Судя
по всему, он отражает некоторую линейную за-
висимость между показателями x и y. При этом
видно, что существуют погрешности измере-
ния, вызванные случайными факторами (а так-
же тем фактом, что измерения значений про-
водились с округлением до целых). Эти по-
грешности несколько искажают общую зависи-
мость, однако в целом вступает в силу закон
58
больших чисел, и искажения в общей массе
сглаживаются. Предположим, что отмеченные
точки выражают реальную ситуацию, которую
необходимо смоделировать.
Дальнейшие вычисления нужно проводить
вручную, поскольку это позволяет задуматься
над тем, что именно и правильно ли Вы делае-
те. Можно порекомендовать проверить полу-
ченные результаты расчетов на компьютере.
Самую трудоёмкую часть работы — построение
доверительных интервалов — разрешается де-
лать сразу на компьютере.
По полученным точкам постройте линейную
модель парной регрессии (y зависит от x) в
полном соответствии с рассмотренным в этой
лекции алгоритмом. Каждый пункт алгоритма
должен сопровождаться выводами по проде-
ланной работе и полученному результату, что
поможет Вам в дальнейшем при разработке об-
щих выводов и рекомендаций по практическо-
му использованию модели.
1422 Кб, 3264x2448
#507 #405266
Проверьте пожалуйста.
12 Кб, 509x249
2472 Кб, 3264x2448
#508 #405416
Я пытаюсь пик понять.

Очевидно левые части двух верхних неравенств складываются и умножаются на среднюю часть третьего с верху неравенства, благодаря чему получаем < epsylon.

И вот у меня выходит правый пик, хотя должен выйти модуль разницы а/б и х/у.
#509 #409364
>>396119
Вы математик? Можно с Вами познакомиться?
#510 #414914
Вопрос. Вот есть в математике доказательство несоизмеримости, теорема Абеля-Руффини, теоремы Гёделя. Касаются они совершенно различных математических объектов, но объединяет их то, что все три, по сути, являются "доказательствами невозможности" - и после их появления, эта самая математика делала резкий уклон в сторону. Иначе, эти доказательства порождали радикально новые пути развития математической мысли.

Есть ли в математике ещё примеры подобных "доказательств невозможности", по фундаментальности и исторической значимости сравнимых с уже упомянутыми?
#511 #415113
Допустим есть конечное множество M, все элементы которого - натуральные числа. Есть известное число N. Нужно выбрать m - подмножество M таким образом, чтобы
1) для любых i, j из m: |i-j| >= N
2) размер m максимален
Есть какие-нибудь быстрые решения без перебора всех вариантов?
Тред утонул или удален.
Это копия, сохраненная 15 июня 2017 года.

Скачать тред: только с превью, с превью и прикрепленными файлами.
Второй вариант может долго скачиваться. Файлы будут только в живых или недавно утонувших тредах. Подробнее

Если вам полезен архив М.Двача, пожертвуйте на оплату сервера.
« /sci/В начало тредаВеб-версияНастройки
/a//b//mu//s//vg/Все доски